05. PDD - Questions

¡Supera tus tareas y exámenes ahora con Quizwiz!

Find the moment of inertia of a solid wood beam 6" wide and 13.5" deep.

1,230 in4 I = = (bd^3) / 12 = (6)(13.5^3)/12 = 1,230 in4

Architectural woodwork for installation in the southwestern United States should have a moisture content of (A) less than 5% (B) 4-9% (C) 5-10% (D) 8-13%

(B) 4-9% The southwestern portion of the US is the driest, so moisture content should approximate the conditions in which the lumber will be used. However, it is difficult to reduce the moisture content much below 5%, so (A) is an unrealistic answer

What is the maximum height that a stair can extend before requiring a landing? (A) 10'-0" (B) 12'-0" (C) 14'-0" (D) 16'-0"

(B) 12'-0" According to Section 1009.10 Vertical Rise of the IBC: "A flight of stairs shall not have a vertical rise greater than 12 feet between floor levels or landings"

According to most building codes, horizontal masonry reinforcement is required every (A) 8" (B) 16" (C) 24" (D) 32"

(B) 16" Most building codes require horizontal reinforcement in both brick and concrete masonry walls a minimum of every 16". The reinforcing may be a continuous truss or ladder type laid in the mortar joints

What is the minimum flow velocity in a wastewater system? (A) 1.5 ft/second (B) 2.5 ft/second (C) 5 ft/second (D) 10 ft/second

(B) 2.5 ft/second Wastewater systems are located to provide gravity flow to treatment facilities or disposal locations. Grades are sloped to provide a velocity between 2.5 feet per second and 10 feet per second

What division would a dry-pipe sprinkler system be located in? (A) 45 (B) 22 (C) 08 (D) 30

(B) 22 Division 01 - General Requirements Division 02 - Existing Conditions Division 03 - Concrete Division 04 - Masonry Division 05 - Metals Division 06 - Wood, Plastics, & Composites Division 07 - Thermal & Moisture Protection Division 08 - Openings Division 09 - Finishes Division 10 - Specialties Division 11 - Equipment Division 12 - Furnishings Division 13 - Special Construction Division 14 - Conveying Equipment Division 22 - Plumbing

According to the ADA/ABA Guidelines, what is the minimum clear floor space for one stationary wheelchair? (A) 24" x 36" (B) 30" x 48" (C) 32" x 48" (D) 60" x 60"

(B) 30" x 48" This critical dimension is the basis for many other accessibility guidelines, such as the amount of clear floor space required at a lavatory and the width of a hallway required to allow two wheelchairs to pass (60", which is 30" times 2)

Which of the following is not true of areas of refuge? (A) Must be at least 30" by 48" (B) Must be at least partially outdoors (C) Must be protected from fire and smoke (D) One is required for every 200 occupants

(B) Must be at least partially outdoors Areas of refuge are not required to be outdoors. They are typically used in interior conditions as a location for disabled persons to wait for assisted rescue.

Where is the ideal location for a fan room? (A) On the ground floor (B) Near the exterior walls (C) In the center of the building (D) Below grade

(B) Near the exterior walls Fan rooms are typically placed near exterior walls because they require direct access to fresh air

In the open-web steel joists structural system, the deep-long span joists of the DLH-series have depths up to (A) 30 in (B) 4 ft (C) 6 ft (D) 10 ft

(C) 6 ft In the open-web steel joists structural system, the deep-long span joists of the DLH-series have depths of up to 6'... spans up to 240' Option (A) is the limit depth for the open-web joists of the K-series Option (B) is the limit depth for open web joists of the LH-series... spans up to 96'

Beyond what decibel level does sound start to be considered harmful? (A) 30 decibels (B) 55 decibels (C) 80 decibels (D) 100 decibels

(C) 80 decibels Sustained sounds at 80 decibels and above can cause permanent hearing loss. Sounds at 125 dB and above cause pain

Which type of tile has the highest moisture absorption rate? (A) Impervious (B) Non-vitreous (C) Semi-vitreous (D) Vitreous

(B) Non-vitreous No tile is truly waterproof but non-vitreous tile has the highest moisture absorption rate. This type of tile is typically not suitable for use in very wet areas such as showers nonvitreous -- 7-15% absorption semivitreous -- 3-7% absorption vitreous -- .05-3% absorption impervious -- 0-.05% absorption

A geotechnical report would most likely be paid for by which party? (A) Contractor (B) Owner (C) Architect (D) Structural engineer

(B) Owner Before an architect can properly begin work a few things are required from the owner: the survey, the geotechnical (or soils) report, and the program

AIAI A201, General Conditions of the Contract for Construction, Article 7, allows the owner to make changes in the work once a construction contract is signed. The present construction environment is such that labor and material prices are changing almost daily. If the owner wants to make changes and cannot agree with the contractor on the cost adjustment to the work, the work may stop. How can the owner make changes to the work without invalidating the contract or stopping the work? (A) The architect can issue a change order (B) The architect can issue a construction change directive (C) The architect can issue minor changes to the work without the approval of the owner or contractor (D) The owner must negotiate with the contractor

(B) The architect can issue a construction change directive A change order requires the owner to accept the cost proposal from the contractor before it can be issued Minor changes to the work are limited to the intent of the contract documents The owner cannot make changes to the work directly through the contractor The architect can issue a construction change directive with only the owner's and architect's signature, requiring the contractor to proceed with the change prior to an agreement on the cost and/or time adjustment

What is a stack vent? (A) The portion of a soil or waste stack that is above the highest branch drain connected to the stack. Its sole function is to vent to the indoors. (B) The portion of a soil or waste stack that is above the highest branch drain connected to the stack. Its sole function is to vent to the outside air. (C) The portion of a soil or waste stack that is above the lowest branch drain connected to the stack. Its sole function is to vent to the indoors. (D) The portion of a soil or waste stack that is above the lowest branch drain connected to the stack. Its sole function is to vent to the outside air.

(B) The portion of a soil or waste stack that is above the highest branch drain connected to the stack. Its sole function is to vent to the outside air.

What is enthalpy? (A) The total conductance of a material (B) The total heat in a system (C) The total moisture in a system (D) The total resistance in a system

(B) The total heat in a system Enthalpy is a thermodynamic quantity equivalent to the total heat content of a system. It is equal to the internal energy of the system plus the product of pressure and volume

A 6" thick concrete slab is reinforced with #4 rebar placed in a horizontal grid and spaced 4" on center. The maximum size of the aggregate shall not exceed (A) 1.5" (B) 1" (C) 2" (D) 2-5/8"

(C) 2" Aggregate shall not be larger than 1/3 of the slab thickness or 3/4 of the minimum space between reinforcing bars, whichever is smaller The slab thickness is 6", so 1/3 of this is 2" The rebar is space 4" apart and is 1/2" in diameter, so the space between the bars is 3.5". 3/4 of this is 2-5/8"

What division would scaffolding be located in? (A) 40 (B) 31 (C) 14 (D) 07

(C) 14 Scaffolding is contained in Division 14 - Conveying Equipment. Temporary scaffolding is contained in Division 01 - general requirements. The type of scaffolding is not defined in this question. It should be assumed the question is referring to permanent scaffolding since Division 01 is not an answer choice

The square footage of the site is 10,000 s.f. An FAR of 2:1 is allowed for this property. What is the maximum building square footage allowed on this site? (A) 5,000 s.f. (B) 10,000 s.f. (C) 20,000 s.f (D) 75,000 s.f.

(C) 20,000 s.f Multiply the available square footage of the site by two

What division would exterior lighting be located in? (A) 12 (B) 37 (C) 26 (D) 32

(C) 26 In the 2004 CSI MasterFormat, Exterior Lighting is listed under Division 26 - Electrical

What is the approximate thickness of single-strength glass? (A) 1/32" (B) 1/16" (C) 3/32" (D) 1/4"

(C) 3/32" Single-strength glass is approximately 3/32″ thick Double-strength glass is approximately 1/8″ thick

A #3 steel reinforcing bar measures what size? (A) 3/32" diameter (B) 3/16" diameter (C) 3/8" diameter (D) 3/4" diameter

(C) 3/8" diameter The # of rebar refers to how many 1/8″ diameter it is. A #3 rebar has a nominal 3/8″ diameter. Likewise #5 has a 5/8″ diameter and so on Rebar generally ranges between #3 and #18

In order to limit the horizontal deflection of a plywood diaphragm system, most building codes limit the span-to-depth ratio of the diaphragm to (A) 2:1 (B) 3:1 (C) 4:1 (D) 5:1

(C) 4:1 Most building codes limit the span-to-depth ratio of a plywood diaphragm to 4:1. This prevents excessive horizontal deflection of the diaphragm. Codes also require that diaphragm deflection be controlled in a way that will not exceed the limits for structural integrity of the diaphragm and the attached load-resisting elements. Excessive horizontal deflection of the diaphragm might lead to damage or failure of the attached resisting elements

A W12x45 beam of A992 steel spans 21 ft. From a table, the total allowable load for a 21 ft span is 61 kips. What is tha maximum load per foot this beam can carry?

2.9 kips/ft w = (61 kips) / (21 ft) = 2.9 kips/ft

What is the maximum length of a corridor with a dead end?

20 feet According to section 1018.1 Dead Ends of the IBC: "Where more than one exit or exit access doorway is required, the exit access shall be arranged such that there are no dead ends in corridors more than 20 feet"

The client mentions that they don't want to pay more than $200 for electricity in any given month. If the average price of electricity is $0.10 per kWh, what is the maximum kWh the building can utilize per month to meet the client's goal?

2000 kWh $200 / $0.10 per kWh = 2000 kWh

A warehouse building is composed of 20'-0" x 20'-0" bays. The roof of the building is supported by joists @ 2'-0" on center that span between girders. The total load on the joist is 100 psf. At each joist-girder point what is the load?

2000 lbs or 2kips Load at each joist = = joist spacing * joist span * load = 2ft * 20' * 100psf = 4,000 lbs The load on the joists is then transferred equally between the two girders it spans 4,000 lbs / 2 = 2,000 lbs

The heat gain for a building has been calculated at 108,000 Btu/hr. A compressive refrigeration machine of ________ tons should be specified. (Fill in the blank.)

9 tons A ton of air conditioning is equivalent to 12,000 Btu/hr. Dividing 108,000 Btu/hr by 12,000 Btu/hr gives 9 tons Note that 12,000 Btu/hr is the amount of refrigeration needed to make one ton of ice per day from 32 F water

The procedure for submitting shop drawings for architectural woodwork is specified in MasterFormat Division (A) 01 (B) 06 (C) 09 (D) 12

(A) 01 Procedures for submittals are found in Division 01, General Requirements. The requirements in individual technical sections refer to Division 01 to define procedural requirements, and each section includes a list of the specific type of samples required for that product

A three-phase motor draws a 9A current at 208V. The power factor is 75%. The power generated is (A) 1.4 kW (B) 2.4 kW (C) 3.6 kW (D) 4.2 kW

(A) 1.4 kW Power, P, is a function of voltage, V, current, I, and the power factor, PF P = = VI(PF) = (208V)(9A)(.075) = 1404 W = 1.4 kW

How much water content must be present in a piece of lumber in order for it to be considered 'seasoned'? (A) 10% (B) 20% (C) 50% (D) 80%

(A) 10%

What division number would a lavatory be located in? (A) 22 (B) 35 (C) 14 (D) 08

(A) 22 Both residential and commercial lavatories are in the Plumbing Fixtures Subdivision of Division 22 - Plumbing

A 4x6 structural steel column is 9'-6" long. The column's least radius of gyration is 1.21". What is approximately the column's slenderness ratio? (A) 94 (B) 113 (C) 135 (D) 162

(A) 94 The slenderness ratio is = = L / r = (9.5 ft)(12 in/ft) / 1.21" = 94.2

A client requested a change order for the demo of existing floor and installation of a new floor in their 500 ft² greenhouse. After seeing the contractor's numbers they've decided to attempt to do the work themselves. How much will the client save by doing the work themselves? Assume the contractor's overhead and profit were an additional 61% on the total. Demolition = $1,150.00 Materials = $490.00 Labor = $1,848.76 Total with Overhead and Profit = $5,616.90

$5,126.90 If the client decides to do the work themselves all they'll need to purchase is the materials at $490.00 To find savings for the client we simply need to subtract the cost of materials from the total. $5,616.90 - $490 = $5,126.90

Installing a sprinkler system in new construction increases the total construction cost by approximately (A) 1.5% (B) 5% (C) 9% (D) 15%

(A) 1.5% According to the NFPA, installing a sprinkler system in a new building adds 1-1.5% to the total construction cost. Retrofitting an existing structure costs considerably more Sprinkler systems can reduce the risk of death and property loss by one-half to two-thirds, and they are encouraged, if not required, in all types of buildings, including single-family residences where more than 80% of fire deaths occur

What division would a metal awning be located in? (A) 10 (B) 04 (C) 28 (D) 05

(A) 10 Awnings are located in Protective Covers under Division 10 - Specialties, in the Exterior Specialties subdivision

For site grading on a project, slopes for permanent fill are not to be steeper than 1 vertical unit for how many horizontal units? (A) 2 (B) 3 (C) 4 (D) 5

(A) 2 Slopes for permanent fill are not to be steeper than 1:2 or 50% slope. The same limit applies to cut slopes for permanent excavations. Deviation from the limit is allowed only with the presentation of a soil report. This limit is expressed in the building code, state transportation agency standards, and accepted practice by civil engineers for typical soil conditions

What kind of joint is used between two concrete slabs in order to maintain alignment after the concrete has cured? (A) A keyed joint (B) A control joint (C) An expansion joint (D) A mortise joint

(A) A keyed joint Joints between two concrete pours (or old and new pours) are always points of weakness. Keyed joints are often used in these situations to provide shear strength

Which type of plastic piping is not used to supply water? (A) ABS (B) CPVC (C) HDPE (D) PEX

(A) ABS ABS piping is used for drainage CPVC is used for hot and cold supply HDPE is used for exterior water supply PE is used for water supply, irrigation sprinklers, and exterior drainage PEX-AL-PEX is used for water supply and compressed air and gas PP is used for industrial supply and waste for chemical resistance and high heat PVC is used for cold water supply

Which of the following are conditions linked to asbestos exposure? (Choose the three that apply.) (A) Asbestosis (B) Brain cancer (C) Lung cancer (D) Mesothelioma (E) Appendicitis

(A), (C), (D) According to the EPA the three major health effects associated with asbestos exposure are: + lung cancer + mesothelioma + asbestosis

Where are dry-pipe sprinkler systems most commonly used? (A) Areas subject to freezing (B) Urban areas (C) Rural areas (D) Smaller buildings

(A) Areas subject to freezing Dry-pipe sprinkler systems are typically used when pipes are subject to freezing such as in a cold climate open parking garage. Dry sprinkler systems are filled with pressurized air or nitrogen. When a valve is triggered the air or nitrogen is released and water is allowed to flow through the pipes and out through the sprinkler heads. This system is complex and the response time until water is flowing can be slow. These two issues can be problematic when a swift fire response is required

Lumber is measured, computed, and priced in __________ (A) Board feet (B) Linear feet (C) Square feet (D) Cubic inches

(A) Board feet Lumber is measured, computed, and priced in board feet. This is equivalent to 144 in³ of material or 12″x12″x1″

What causes "hard" water? (A) Calcium and magnesium salts dissolving in water (B) Natural oils failing to emulsify when mixed with water (C) Weak acids dissolving in water (D) Weak bases dissolving in water

(A) Calcium and magnesium salts dissolving in water Hard water is caused by calcium and magnesium salts dissolving in water. Studies are inconclusive on whether drinking hard water is harmful, but it is certainly a problem in industrial facilities where mineral buildup can cause costly breakdowns and repairs

What is the unit for candlepower? (A) Candela (B) Lumen (C) Lux (D) Watt

(A) Candela The unit for candlepower is the candela (cd). Candela is the base measurement for describing luminous intensity. It tells you how bright the light source is which shows how far away from an object you can be and while still being able to see it Lux (lx) measures illuminance, which is the amount of light on a surface per unit area. A single lux is equal to one lumen per square meter The lumen (lm) is a measurement of luminous flux, or the total amount of visible light. To put it simply, the lumen rating is how much total visible light is produced by a light source A good way to remember the differences between terms is: + Lumens are how much light is given off + Lux is how bright your surface will be + Candela measures the visible intensity from the light source

What is the term for lengthwise wood grain separation caused by seasoning? (A) Check (B) Pitch pocket (C) Shake (D) Split

(A) Check Lengthwise wood grain separation caused by seasoning is termed a check A shake is lengthwise grain separation inherent in the wood before seasoning

The most appropriate strategy to minimize conflicts between engineering and architectural documents is to (A) Conduct regular coordination meetings (B) Send the drawings to the owner for review (C) Use in-house engineers (D) Schedule peer review of documents

(A) Conduct regular coordination meetings The best way to minimize conflicts and discrepancies between the architect and consultant drawings is to keep everyone in the loop. Setting aside time to coordinate drawings is the best practice to ensure everyone is on the same page. The best way to do that is with face-to-face meetings where everyone is fully present

What is the term for the horizontal or vertical joints between two successive concrete pours? (A) Construction joints (B) Concrete joints (C) Control joints (D) Expansion joints

(A) Construction joints Construction joints are the horizontal or vertical joints between two successive concrete pours These joints are often keyed to provide additional shear strength

Concrete strength __________ as extra water required for workability increases (A) Decreases (B) Stays the same (C) Increases (D) Fluctuates

(A) Decreases Concrete strength decreases as extra water required for workability increases. During construction it is always important to find a balance of workability without compromising the concrete strength by adding too much water

What is the ratio of the ultimate strength of a material to its working stress? (A) Factor of safety (B) Factor of stress (C) Factor of strength (D) Modulus of elasticity

(A) Factor of safety Factory of safety is the ratio of (yield stress) / (working stress). This is the same as the ultimate strength of a material to its working stress

Which of the following is generally most expensive? (A) Field labor (B) Shop labor (C) Structural labor (D) All of the above cost roughly the same

(A) Field labor Field labor is generally most expensive because it involves the most varying conditions

The k value for a material is 0.13. If it's thickness is 8" what is its U-value?

0.01625 First calculate the R value R = = x / K = 8" / .13 = 61.54 Convert R to U U = = 1 / R = 1 / 61.54 = 0.01625

A project is located in a climate where severe frost is a concern. The client is adamant they would like a brick veneer to contrast the snowy backdrop. Which mortar type is most appropriate to use for the veneer wall? (A) M (B) S (C) N (D) O

(A) M Type M mortar is high-strength mortar. It is suitable when masonry is subject to high lateral loads, compressive loads, severe frost, or masonry below grade. It should be specified for exterior applications at or below grade Type N or S is the best for exterior applications above grade and for interior load-bearing walls Type O is well suited for interior and protected exterior non-load-bearing partitions

Which statement about designing for wind forces is correct? (A) Maximum drift should be limited to 1/500 of a building's height (B) Maximum drift should be limited to 1/100 of a building's height (C) Drift between adjacent stories should be limited to 0.025 times the story height (D) Drift between adjacent stories should be limited to 0.0025 time the story height

(A) Maximum drift should be limited to 1/500 of a building's height

Lights with a lower Kelvin color temperature would be which color? (A) Red (B) Yellow (C) Green (D) Blue

(A) Red Color temperature is a way of rating white light and is based on the theoretical temperature and color emitted. "Cooler" (more blue) light has a higher Kelvin "Warmer" (more red) light has a lower Kelvin

An area/volume estimate is primarily used during which phase of a project? (A) SD (B) DD (C) CD (D) CA

(A) SD Area/volume estimate is a type of cost estimating, used primarily during the schematic design (SD) phase because of the lack of details available, to determine a preliminary cost estimate based on cost per square foot (area) and cost per cubic foot (volume)

Which of the below soil types is the most thermally conductive? (A) Saturated sand (B) Saturated silt (C) Sand (D) Clay

(A) Saturated sand Thermal conductivity (BTU/FT hr °F) Sand: 0.44 Clay: 0.64 Saturated silt or Saturated clay: 0.96 Saturated sand: 1.44

By what method is copper pipe joined? (A) Soldering (B) Welding (C) Solvent glued (D) Taping

(A) Soldering

The type of project estimate that deals with a project's functional units or assemblies is called what? (A) Subsystems estimate (B) Area/Volume estimate (C) Detailed estimate (D) Mechanical estimate

(A) Subsystems estimate The correct answer is subsystem estimate. Area/volume is less detailed, and a detailed estimate is more detailed. Mechanical estimates do not exist

What is 'specific heat'? (A) The number of Btus required to raise the temperature of a specific material by 1°F (B) The number of joules required to raise the temperature of a specific material by 1°F (C) The number of cals required to raise the temperature of a specific material by 1°F (D) The number of watts required to raise the temperature of a specific material by 1°F

(A) The number of Btus required to raise the temperature of a specific material by 1°F

What is dew point? (A) The temperature at which which water droplets begin to condense (B) The adiabatic saturation temperature (C) The air temperature (D) The measurement at which air is considered humid

(A) The temperature at which which water droplets begin to condense Dew point is the atmospheric temperature (varying according to pressure and humidity) below which water droplets begin to condense and dew can form

Incandescent lighting tends to give off what color light? (A) Yellow (B) Blue (C) Red (D) White

(A) Yellow Incandescent lighting is known to give off yellowish light. When this yellowish light hits surfaces of cool colors it tends to dull them. When it hits surfaces of warm colors, it tends to brighten them

Which of the following statements about specifications are true? (Choose the two that apply.) (A) Both narrowscope and broadscope sections can be used in the same project manual (B) Drawings are more binding than specifications if there is a conflict (C) Specifications show quality; drawings show quantity (D) Proprietary specifications encourage competitive bids (E) Specifications should not be open to interpretation if they are the "base bid with alternates" type (F) Proprietary specifications are the most difficult for an architect to write

(A) and (C) Both narrowscope and broadscope sections can be used in the same project manual Specifications show quality; drawings show quantity

A fire-rated gypsum board partition must (A) be Type X gypsum board (B) stop at the ceiling (C) be attached with screws (D) be non-bearing

(A) be Type X gypsum board Fire-rated partitions must be constructed according to tested and approved methods that include using Type X gypsum board, the method of attachment to the framing, how the joints are finished, the type and size of studs, and other details. In addition, the fire separation must extend from the slab to the rated slab above, not just to a suspended, finish ceiling Fire-rated partitions can be bearing or non-bearing

A project's contract documents require concrete samples to be tested. Who makes the necessary arrangements with the appropriate testing agencies? (A) contractor (B) owner (C) architect (D) structural engineer

(A) contractor Because the testing is required by the building code and is included in the contract documents, it is the contractor who is responsible for making arrangements with the acceptable testing agencies to test the concrete samples

Which of the following types of conduit is best suited for use when connecting to a motor? (A) flexible metal conduit (B) electric metallic tubing (C) intermediate metal conduit (D) rigid steel conduit

(A) flexible metal conduit Flexible conduit should be used to connect to a motor because it can "give" with the movement of the machinery. It is a good choice for any location where there is vibration or where it is impossible to use a straight run of rigid conduit

Stress in a body (A) is an internal resistance to deformation (B) is the change in size do to force (C) is the tendency of the body to rotate (D) happens where the moment is zero

(A) is an internal resistance to deformation Stress in a body is the internal resistance to deformation or to the action of the external force

When selecting a fire extinguisher cabinet, the most critical design feature is the (A) projection distance from the wall (B) size of the glazing (C) height of the cabinet enclosure (D) finish

(A) projection distance from the wall The ADA/ABA Guidelines limit the projection of any construction element to a maximum of 4" from a wall when the element is located between 27" and 80" above the floor In addition to meeting the 4" projection requirement, the top of the cabinet must not be more than 5' above the floor Glazing and finish are not important. however, local jurisdictions may have requirements for the color and lettering type used for identification

The goal of lighting protection system is to (A) provide a continuous path to the ground for a lightning strike (B) prevent a lightning strike (C) prevent damage to computer equipment (D) attract lightning

(A) provide a continuous path to the ground for a lightning strike The best form of lightning protection is to route the energy from the strike to a point where the strike is least likely to damage mechanical, electrical, and computer systems or cause a fire

Which of the following statements about detectors is true? (A) smoke detector covers should be used in areas under construction (B) Ionization detectors should be installed in a restaurant kitchen (C) Smoke detectors may be installed in ductwork in lieu of installation in occupied areas of a building (D) Spot heat detectors can detect a fire before any other type of detectors

(A) smoke detector covers should be used in areas under construction During construction, covers should be used to protect detectors from damage, accumulating dust that could destroy the sensors, and from sounding a false alarm Ionization detectors would be a poor choice for a restaurant kitchen or any other location that tends to be hot, because of the high likelihood of false alarms Smoke detectors are installed in ductwork to direct the HVAC system to shut down in case of a fire, so that smoke and other toxic gases are not transported throughout the building Spot heat detectors would detect a fire last, and after the most deadly smoke stage

Energy transfer wheels (A) temper incoming air with exhaust air (B) use ground temperature for heating or cooling (C) capture the heat of flue gases to warm cool water (D) exchange heat from solar panels to an airstream

(A) temper incoming air with exhaust air An energy transfer wheel transfers heat between two airstreams (incoming air and exhaust air) using a lithium chloride-impregnated heat exchanger. These devices transfer both latent and sensible heat Option (B) describes a ground-source heat pump. This type of system uses the constant temperature of the ground to heat or cool a building Option (C) describes a boiler fuel economizer Option (D) describes a standard heat exchanger

A subsystems estimate is primarily used during which two phases of design? (A) SD (B) DD (C) CD (D) CA

(A), (B) Subsystems estimate is a type of cost estimating, used during the schematic design (SD), and design development (DD) phases. Subsystem estimates deal with a project's functional units or assemblies and enable comparisons between different conceptual solutions. Estimates of this type include major systems (foundations, superstructure, roofing, fixed equipment etc.)

Which of the following are methods used to detect air infiltration? (Choose the two that apply.) (A) Air change method (B) Crack method (C) Critical heat method (D) Design day method (E) Time-sensitive method

(A), (B) The two methods used to detect air infiltration are the air change method and the crack method. Air infiltration is the seepage of air through openings in the building envelope. The air change method measures how often the air in a building is replaced with new air. The crack method uses the total length of cracks or joints to estimate how much air infiltration there is

Which of the following plastic types are rigid? (Choose the three that apply.) (A) ABS (B) CPVC (C) PP (D) PE (E) PEX-AL-PEX

(A), (B), (C) The following plastic pipes are flexible PEX-AL-PEX, PE The following plastic pipes are rigid ABS, CPVC, PP, PVC HDPE may be rigid or flexible

Air barriers are designed to stop infiltration and exfiltration caused by which of the following? (Choose the three that apply.) (A) wind pressure (B) stack pressure (C) HVAC fan pressure (D) vapor pressure (E) pressure in an elevator shaft (F) pressure in a built-up roofing system

(A), (B), (C) Wind pressure, stack pressure, and HVAC fan pressure can all influence infiltration and exfiltration rates Vapor pressure does not cause air movement; rather, vapor pressure is a movement of moisture Pressure in an elevator shaft does not influence air barriers in an exterior wall

Which of the following is a recommended connection for a three-phase transformer? (Choose the three that apply.) (A) delta to delta (B) delta to wye (C) wye to delta (D) wye to wye (E) alpha to delta (F) delta to alpha

(A), (B), (C) Wye-to-wye connections are rarely used because they can cause problems with harmonics and can interfere with communications systems within a building

Which construction types are required to have non-combustible exterior wall assemblies? (Choose all that apply.) (A) Type I (B) Type II (C) Type III (D) Type IV (E) Type V

(A), (B), (C), (D) Type I and II construction consists of building elements that are non-combustible. Type III construction has non-combustible exterior walls and combustible or non-combustible interior walls. Type IV (HT or heavy timber) has noncombustible exterior walls and heavy timber interior elements. Type V construction has combustible or noncombustible building elements It should be noted that combustible/noncombustible are not that same as protected/unprotected which are clarified with an A or a B after the type of construction (Type I-A or Type I-B)

What is true of the plastics used in place of glass in architectural applications? (Choose the four that apply.) (A) It can easily be cut, bent, and formed (B) It has a high coefficient of thermal expansion (C) It is not as hard as glass (D) It is normally less expensive than glass (E) It is more resistant to breakage than glass (F) It scratches less easily than glass does

(A), (B), (C), (E) Plastic that replaces glass may be advantageous to use because it can be easily cut, bent, and formed and is more resistant to breakage However plastic does have a higher coefficient of thermal expansion so proper detailing is needed to allow for this without compromising the envelope. Oddly enough, it also tends to be more expensive than glass despite it's often inferior appearance. Furthermore plastic is softer than glass so it is more susceptible to scratches (but not breaking)

Which of the following are parts of the contract documents? (Choose the four that apply.) (A) an addendum (B) a change order (C) special supplementary conditions (D) the contractor's bid (E) a written amendment signed by the owner and contractor (F) instructions to bidders

(A), (B), (C), (E) The contract documents form the contractual relationship between the owner and contractor and are defined in AIA A201 "The Contract Documents are enumerated in the Agreement between the Owner and Contractor, and consist of the Agreement, Conditions of the Contract, Drawings, Specifications, Addenda issued prior to the execution of the Contract, other documents listed in the Agreement, and Modifications issued after execution of the Contract..."

Which of the following are common types of asbestos? (Choose the three that apply.) (A) Amosite (B) Chrysotile (C) Crocidolite (D) Sugilite

(A), (B), (D) The following are types of asbestos: Actinolite Amosite Anthophylite Chrysotile Crocidolite Tremolite

Which statements about drainage are correct? (Choose the three that apply.) (A) The vent stack connects with the stack vent above the highest fixtures served by the stack (B) The vent stack extends through the roof (C) Vents help prevent the drainage of water from traps (D) The house drain cannot also serve as the building sewer (E) Cleanouts are always a necessary part of a drainage system (F) Vents have a maximum size of 1"

(A), (B), (D) The minimum slope of drains depends on the size of the pipe. The vent stack may extend through the roof, but this is not required. In many cases, the vent stack connects with the stack vent above the highest fixture served by the stack The building sewer begins 3' outside of the building's exterior wall

Which of the following receptacle locations would require ground fault circuit interrupter (GFCI) protection? (Choose the four that apply.) (A) an exterior receptacle for holiday lights (B) a coffeemaker receptacle on a kitchen counter (C) a receptacle located in a finished living area in a basement (D) a receptacle for a chest freezer located in a garage (E) a receptacle next to a bathroom lavatory (F) a receptacle in an upstairs bedroom

(A), (B), (D), (E) GFCI protection is available on both receptacles and breakers. The devices constantly monitor the amount of electricity flowing through a circuit. If the GFCI detects any variation, it instantly shuts off power to that circuit. The person may still be shocked but will not be electrocuted

Which of the following are required of lavatories to meet ADA Accessibility Guidelines? (Choose the four that apply.) (A) The lavatory rim shall be no more than 34" above the finish floor (B) Clear floor space of 30"x48" must be provided in from of the lavatory (C) All piping must be concealed within cabinetry (D) The bowl of the sink must be no greater than 6/5" deep (E) A minimum vertical clearance of 29" must be provided in front of the lavatory (F) All piping must be insulated

(A), (B), (D), (E) Hot water and drain pipes must be protected with an enclosure, wrapped with insulation or plastic covers, or concealed within cabinetry so that a disabled person cannot come in contact with them and be burned

When incorporating the mechanical engineer's specifications into the project manual, the architect should establish which of the following? (Choose the four that apply.) (A) exact numbering system of the specification sections (B) type of header and footer used on each page (C) content of each specification section (D) page layout of the specification sections (E) type of mechanical equipment to be specified (F) where the cooling tower should be located

(A), (B), (D), (E) The architect is responsible for developing the overall format and appearance of the project manual and the specification sections. Each consultant is responsible for the content of their respective specifications. The location of the cooling tower is found on the drawings

Which of the following metals are considered anodes? (Choose the two that apply.) (A) Cadmium (B) Gold (C) Magnesium (D) Silver (E) Stainless steel (F) Titanium

(A), (C) According to Olin's Construction Principles, Materials, and Methods, the electrolytic solution potentials in volts for metals are as follows: Magnesium: -1.73 Zinc: -1.00 Aluminum: -0.83 Cadmium: -0.82 Mild Steel: -0.58 Lead: -0.55 Tin: -0.49 Titanium: -0.30 Brass: -0.28 Copper: -0.20 Stainless Steel: -0.15 Silver: -0.15 Gold: -0.00 The closer the voltage potential is to 0.00 the more cathodic each metal is. The further from 0, the more anodic each metal is

Which statements about a passive solar water heating system are correct? (Choose the four that apply.) Such a system has a... (A) storage tank that is placed above the collector (B) low initial cost (C) high operation cost (D) reliable mechanical system (E) pump (F) storage tank that can cause structural costs

(A), (B), (D), (F) Because a passive solar water heating system has no pump, gravity is used to circulate the water, and the storage tank must me placed above the collector In general, a passive solar water heating system has both a low initial cost and a low operation cost A passive solar water hearing system has a high mechanical reliability because it has no pump that can fail or malfunction Placing the storage tank on the roof can cause structural problems if it is heavy and the roof framing is inadequate

Which of the following statements about tilt-up walls is correct? (Choose the four that apply.) (A) Tilt-up walls are reinforced concrete walls that are precast generally in a flat position and later tilted up to a vertical position (B) Tilt-up walls are subject to high stresses during construction (C) Exterior columns used with tilt-up walls must be precast concrete columns (D) Tilt-up walls provide a good fire resistance, great strength, and low maintenance (E) Tilt-up walls are delivered on a truck (F) Tilt-up walls can be supported by a structural steel frame

(A), (B), (D), (F) Exterior columns used with tilt-up walls do not have to be precast concrete Tilt-up walls can be cast on-site

What information is needed by building departments? (Choose the three that apply.) (A) construction documents (B) site plans (C) fire protection shop drawings (D) structural calculations (E) structural drawings (F) occupant load calculations

(A), (B), (E) Construction documents Site plans Structural drawings

Which of the below building elements are most likely to affect plenum depth? (Choose the three that apply.) (A) Duct size (B) Structural beams (C) Finish floor (D) Return air vent (E) Electrical conduit

(A), (B), (E) Of the listed items, structural beams, electrical conduit, and duct size will mostly affect plenum depth Finish floors are located outside of the plenum Most return air vents are approximately the same depth as the suspended ceiling itself. While this may technically make the plenum fractions of an inch larger, it is not a typical component in determining the required depth of a plenum. The thickness of the suspended ceiling itself is used instead. This answer is not correct because there are other more appropriate answers

Which statement about heat pumps are true? (Choose the four that apply.) (A) Heat pumps use the refrigeration cycle to heat and cool (B) Heat pumps can be connected to solar collectors and water storage tanks (C) Air-to-air heat pumps are appropriate for areas with cold winters (D) Heat pumps have a relatively low initial cost (E) Air-to-air heat pumps are the most common types of pumps used for small buildings (F) Heat pumps minimize operating costs in moderate climates

(A), (B), (E), (F) Air-to-air heat exchangers reclaim waste energy from the exhaust air stream and use it to condition the incoming fresh air. The coefficient of performance (COP) of an air-to-air heat pump decreases as the temperature approaches the freezing mark, so it is not appropriate for colder climates Air-to-air heat pumps generally have high initial cost compared to evaporative cooling or through-the-wall units. Heat pumps can both heat and cool by transferring heat from one place to another. To extend their efficiency, heat pumps can be connected to either solar energy systems or water storage tanks. Heating or cooling incoming air needs the use of a reversible refrigeration cycle, which is very economical to operate. Heat pumps are the most common type of pumps in small buildings because they are the most efficient and economical to operate

Which of the following are types of concrete joints? (Choose the four that apply.) (A) Construction (B) Control (C) Dual-ended (D) Expansion (E) Extension (F) Isolation (G) Mortise

(A), (B), (E), (F) Mortise joints are used in wood construction Dual-ended and extension joints are fictional terms

What are the most common steel structural systems? (Choose the two that apply.) (A) Beam-and-girder system (B) Closed-web steel joist system (C) Open-web steel joist system (D) Type A steel framing system

(A), (C) The most common steel structural systems are the beam-and-girder system and open-web steel joist system

Which of the following strategies could be used to address reentrant corners in designing a building mass? (Choose the three that apply.) (A) Seismic joints (B) Discontinuous shear wall (C) Splayed Corners (D) Stronger connection at the corners (E) Cross bracing (F) Discontinuous horizontal diaphragm

(A), (C), (D) In general there are two basic solutions to addressing reentrant corners: 1 - strengthening the corners 2 - separating the building into less complex geometries Using seismic joints effectively separates the building into separate geometries Stronger connections and splayed (or chamfered) corners strengthen the corners

A successful detail should account for which 3 of the following? (A) Construction trade sequence (B) Safety of installation (C) Tolerances (D) Clearances (E) Material alternates

(A), (C), (D) Successfully drawn details should display the proper construction trade sequence, tolerance, and clearances. Safety of installation is up to the contractor. The architect is not responsible for providing alternate materials for the contractor to select from

Two adjacent rooms are separated by an acoustical partition. There is a noise-producing source in one of the rooms (Room B). The noise reduction to the other room (Room A) is dependent on which of the following? (Choose the three that apply.) (A) area of the partition (B) thickness of the partition (C) transmission loss of the partition (D) absorption of surfaces in Room A (E) noise source in Room B (F) floor area of Room A

(A), (C), (D) The thickness of a partition or other acoustical separation is irrelevant to the total noise reduction within a space. Factors which would influence the effectiveness of the partition are transmission loss, the area of the barrier, and the total sound absorption within the "quiet" space The formula for calculating the noise reduction is NR = TL + 10 log A/S NR = noise reduction, in decibels (dB) TL = transmission loss level of the barrier (dB) A = total acoustical absorption of the receiving room, in sabins (ft2) S = area of the barrier, in ft2

Which of the following is a part of the contract documents? (Choose the four that apply.) (A) an addendum (B) bidding documents (C) general conditions of the contract for construction (D) change orders (E) specificaitons (F) instruction to bidders

(A), (C), (D), (E) The contract documents form the contractual relationship between the owner and contractor and are defined in AIA A201 "The Contract Documents are enumerated in the Agreement between the Owner and Contractor, and consist of the Agreement, Conditions of the Contract, Drawings, Specifications, Addenda issued prior to the execution of the Contract, other documents listed in the Agreement, and Modifications issued after execution of the Contract..."

Which of the following plaster types are basecoat plasters? (Choose all that apply.) (A) Wood-fibered plaster (B) Molding plaster (C) Bond plaster (D) Gauging plaster (E) Ready-mixed plaster (F) Gypsum keene's cement plaster

(A), (C), (E) Basecoat platers from the available options are + wood-fibered plaster + bond plaster + ready-mixed plaster

The architect's client is a coffee company called Copper Cup. They are adamant that all of the signage on the building be made out of copper. A parking identification sign which says "HANDICAP STALL", is put up for the only accessible stall required on the lot. The 18"Hx12"W sign is made of copper, is mounted at 60"H (measured to the bottom), and is fixed to a galvanized steel post with aluminum bolts. What issues should the architect be worried about? (Choose the three that apply.) (A) The sign must include the international symbol of accessibility (B) The sign mounting height is incorrect (C) The aluminum bolts are likely to corrode (D) The copper sign is likely to corrode (E) The parking sign should identify the space as van accessible (F) The sign must be 20"Hx18"W

(A), (C), (E) The architect should worry about galvanic action between the copper and aluminum. Aluminum has a lower voltage potential than copper, so when water or moisture is added, the aluminum bolts are likely to corrode According to the 2010 ADA standards, an identification sign which includes the international symbol of accessibility should be at every accessible stall (ref. 2010 ADA §502.6) Furthermore, for every 6 or fraction of 6 ADA stalls one is required to be van accessible (ref.e 2010 ADA § 208.2.4) Since there is only one ADA stall in this parking lot, it is required to be van accessible. Van accessible stalls must be identified as such (ref. 2010 ADA §502.6) The size of the sign is not restricted as long as the characters can fit at the sizes designated in the ADA code. The sign mounting height of 60″ measured to the bottom is correct (ref. 2010 ADA §502.6)

A project design team for a speculative office building owned by a real estate development company is in the construction document phase. The development company informs the architect that the project has been sold to a software development company. The new owner intends to be the sole tenant and to make this new building the corporate headquarters for the company. How will this change impact the completion of the design and construction documents? (Choose the four that apply.) (A) Mechanical and electrical systems must be redesigned and redocumented (B) On-site parking must be redesigned (C) Structural design for the building can proceed with construction documents (D) Site drainage must be redesigned (E) The elevator consultant must redesign the elevator controls (F) The fire protection system must be redesigned

(A), (C), (E), (F) Because the use of the building remains as an office, the structural design, site drainage, and on-site parking will remain unchanged

The air pressure going through the air ducts needs to be increased. What are potential solutions to increase the pressure? (Choose the three that apply.) (A) Reduce duct size (B) Reduce fan size (C) Increase the duct length (D) Increase the fan size (E) Increase the duct size

(A), (D) Decreasing the size of the ducts and increasing the size of the fan will both increase the air pressure within an HVAC system Larger ducts, longer duct lengths, and smaller fans will reduce the pressure

The design team has completed the design development documents for a new two-story tenant commercial office building on a site zoned with a height limit of three stories. As the team begins the construction document phase, the owner announces to the team that the project has been changed to include a third story to the building. Which of the following is included in an amendment to the design contract? (Choose the two that apply.) (A) site planning (B) floor plan core elements (C) building setbacks from the property line (D) facade design (E) door hardware (F) roofing system

(A), (D) Site planning will be altered to accommodate the additional parking The facade will be redesigned to add the third floor

Which of the following are included in a typical transformer specification? (Choose the four that apply.) (A) sound level in decibles (B) the type of cooling system (C) physical dimensions (D) voltage (E) insulation class (F) kVA rating

(A), (D), (E), (F) Transformer specification should include the type of transformer, phase, voltage, kVA (kilovolt-ampere) rating, sound level, and insulation class It is unlikely that the physical dimensions of a transformer would be included in the specification, as the size tends to be determined by the technical criteria, particularly by the capacity and insulation type. The size of the units can vary greatly from one manufacturer to the next

An architect is designing an outdoor space and would like to specify wood for any finishes, beams, and columns. Which types of wood are appropriate to consider? (Choose all that apply.) (A) Teak (B) Bamboo (C) Douglas Fir (D) Redwood (E) Maple (F) White Oak (G) Ipe

(A), (D), (F), (G) The key to this question is knowing which type of wood has resistance to rot and it's lifespan. Teak has a long lifespan of greater than 25 years and has excellent insect resistance, though some species are susceptible to marine borers Bamboo has a short lifespan of less than 5 years and has poor insect/borer resistance Douglas Fir a moderate lifespan of between 10-15 years but has poor insect/borer resistance Redwood has a lifespan starting from 10-25+ years. The wood becomes more durable the older the tree is. Maple has a short lifespan of less than 10 years and has poor insect/borer resistance White Oak has a long lifespan of greater than 25 years and is often used in boatbuilding. In contrast, Red oaks have a short lifespan of less than 5 years and poor insect/borer resistance Ipe has a long lifespan of greater than 25 years and has excellent insect resistance, though some species are susceptible to marine borers

The owners of a residential project plan to shop for and purchase all the kitchen appliances in their new home. They want the contractor to be responsible for coordinating the electricity, gas, and plumbing, and for installing the equipment. how should these items be noted on the drawings? (A) "N.I.C." (B) "owner furnished-contractor installed" (C) "cash allowance" (D) by name, as "range," "refrigerator," and so on

(B) "owner furnished-contractor installed" NIC = not in contract Cash allowances are used when an owner wishes to include an item in the contract but has not yet selected something to be specified If the items were noted by name, as "range," "refrigerator," and so on, it would imply that the contractor is responsible for providing, coordinating, and installing the equipment

What division would a precast concrete tee be located in? (A) 08 (B) 03 (C) 21 (D) 16

(B) 03 Division 01 - General Requirements Division 02 - Existing Conditions Division 03 - Concrete Division 04 - Masonry Division 05 - Metals Division 06 - Wood, Plastics, & Composites Division 07 - Thermal & Moisture Protection Division 08 - Openings Division 09 - Finishes Division 10 - Specialties Division 11 - Equipment Division 12 - Furnishings Division 13 - Special Construction Division 14 - Conveying Equipment

Calculate the equivalent thickness of a 12" concrete block that is 75% solids (A) 8.5" (B) 8.75" (C) 9" (D) 9.25"

(B) 8.75" The equivalent thickness is a measurement of the amount of concrete in a hollow core block. It is equivalent to the thickness that the block would be if it were the same height and length but cast without holes. This information is often necessary for calculating fire resistance of assemblies A nominal 12" concrete block is actually 11-5/8" thick. Multiply the actual thickness by the percentage of solids to arrive at equivalent thickness Equivalent thickness = = (.75) (11-5/8") = 8.75"

What is a luminaire? (A) Filament used in a bulb (B) A complete light fixture assembly (C) an integrated transformer within a light fixture (D) the light bulb within a light fixture

(B) A complete light fixture assembly A luminaire is a complete electrical unit. It is composed of a ballast, housing, reflector, lamp, baffle, mounting frame, lens or diffuse, and trim

What site would be best suited for a mat slab or pile foundation? (A) A steep cliff-like site (B) A site with a high water table (C) A flat site (D) A site near a lake

(B) A site with a high water table Mat and pile foundations would be appropriate solutions where there is a high water table. The main idea behind using these two types of foundations in this condition is so the structural integrity of the foundation is not compromised. Since a mat foundation is shallow, it does not come into contact with the water table A pile foundation would extend past the water table to be supported below it

Which of the following HVAC system types requires the least amount of space? (A) All-air system (B) All-water system (C) Air-water system (D) All of the above require about the same amount of space

(B) All-water system All-water systems are smaller since water can travel through much smaller pipes All-air systems are largest because of the size of ductwork and size of the mechanical equipment needed to circulate the air

Where are expansion joints NOT required? (A) At additions to existing buildings (B) At corners of buildings (C) At joints of building wings (D) In buildings more than 200 feet long

(B) At corners of buildings Expansion joints are required for large buildings or buildings without rectangular geometries Expansion joints are not required at corners of buildings

What insulating product is composed of recycled newspaper or other paper products? (A) Wool Batt (B) Cellulose (C) Fiberglass Batt (D) Spray foam

(B) Cellulose Cellulose insulation is generally composed of recycled paper products although technically speaking it can come from any cellular plant source. The word cellulose comes from the french word 'cellule' which means 'small cell'

What is the most severe flame spread rating? (A) Class A (B) Class C (C) Class D (D) Type 1 (E) Type 5

(B) Class C The Life Safety Code (NFPA 101)and Section 803.1 of the International Building Code limit finishes for interior walls and ceilings to materials in three classes (A, B, or C, with A being the lowest flame spread and C being the highest) and gives greater restrictions for certain rooms

Which of the following is NOT typically used to fill the air space between the veneer and tieback wall in a cavity wall? (A) Air (B) Concrete (C) Cellulose (D) Fiberglass insulation (E) Grout

(B) Concrete It is typical for the cavity in a masonry cavity wall to be filled with air, insulation (which includes fiberglass & cellulose), or grout A grouted wall would be used in conditions where a more substantial structural wall is required. The grout in the cavity is accompanied by reinforcing bars. This wall is considered a reinforced grouted masonry wall

Which type of joint is used where two separate placements of concrete meet and where reinforcement may be continuous? (A) Cold (B) Construction (C) Control (D) Isolation

(B) Construction Cold joints are used when a concrete slab is too large to be completed in a single concrete pour Construction joints are used where two separate placements of concrete meet and where reinforcement may be continuous Control joints (contraction joints) are tooled, sawed, or formed grooves in the concrete to create weak points where cracking can occur Isolation joints are used where adjacent areas of concrete may move in three directions and where the formation of cracks is to be avoided

Stipulated sum compensation is also known as what? (A) GMP (B) Fixed fee (C) Cost+fee (D) Percentage of construction cost

(B) Fixed fee Stipulated sum is also known as fixed fee or lump sum

The zoning ordinance within the jurisdiction your project is constructed, gets updated. The update throws your building into non-compliance, however your building is allowed to remain as designed. Your building has been what? (A) Rolled-in (B) Grandfathered (C) Aggrieved (D) Annexed

(B) Grandfathered Your project has been "grandfathered" into the new code as a non-conforming use. It's permitted to remain as constructed as long as no substantial changes are made to the project

Life Cycle Assessments follow a standard distributed by which organization? (A) ASHRAE (B) ISO (C) IBC (D) ASTM

(B) ISO Life Cycle Assessment (LCA) follows standards put out by the International Organization for Standardization (ISO)

Typically, a bond beam is constructed of what? (A) Glass (B) Masonry (C) Steel (D) Wood

(B) Masonry A bond beam is a horizontally reinforced concrete or concrete masonry beam built to strengthen and tie a masonry wall together. A bond beam is often placed at the top of a masonry wall with continuous reinforcing around the entire perimeter

A single-wythe wall is composed of what type of material? (A) Concrete (B) Masonry (C) Steel (D) Wood

(B) Masonry Wythes are a type of masonry construction. Wythes are classified as single, double, or cavity

For earth of a depth between 10' and 20', the most effective type of retaining wall is (A) gravity wall (B) cantilever wall (C) a counterfort wall (D) either a gravity or a counterfort wall

(B) cantilever wall A gravity wall is not economical beyond a depth of about 4' A counterfort wall is a stronger retaining wall that is generally used for depths of about 25' or more

The following are pages in a set of construction drawings for a small conference room renovation in a large office building: electrical and data plan, finish schedule, demolition and new work plans (on the same sheet), reflected ceiling plan, and interior elevations. If the drawings are arranged properly in a set, which drawing from the list will be the third drawing to be placed in the set? (A) electrical and date plan (B) interior elevations (C) demolition and new work plans (same page) (D) reflected ceiling plan

(B) interior elevations

It would be appropriate for an architect to specify a mockup for (A) a concrete masonry unit (CMU) foundation wall for a small warehouse (B) kitchen cabinetry, granite countertops, and mosaic tile backsplashes for a series of identical apartments (C) a custom-designed carpet inlay in the lobby of a hotel (D) a portion of a brick wall for an addition to a 1760s building

(B) kitchen cabinetry, granite countertops, and mosaic tile backsplashes for a series of identical apartments A mock-up is a full-scale preview of a building assembly. Mock-ups can be very expensive, so it is important to specify them only where they are truly necessary. Approved mock-ups can often be integrated into the work, making them more cost-effective In this problem, the best application of mock-up would be the apartment kitchen. In the case of the cabinetry, the mock-up would allow the architect and owner to see the kitchen in place and sign off on it before the contractor orders the materials for the other apartments. Modifications could be made relatively inexpensively based upon the mock-ups, as opposed to waiting until all of the cabinets and countertops have been fabricated to make a change

Where is a vapor barrier typically placed within a wall? (A) on the cool side (B) on the warm side (C) on the interior side (D) on the exterior side

(B) on the warm side The correct answer is on the warm side. Without additional climatic information, it's not possible to determine if the vapor barrier should be on the exterior or interior side

What is the full name for PVC piping? (A) chlorinated polyvinyl chloride (B) polyvinyl chloride (C) polyethylene vinyl chloride (D) polyethylene vinyl carbon

(B) polyvinyl chloride

What are the primary types of water supply systems? (Choose the two that apply.) (A) Static feed system (B) Upfeed system (C) Direct upfeed pumping system (D) Downfeed system (E) Tankless system

(B), (D) The primary types of water supply systems are upfeed and downfeed A direct upfeed pumping system, also known as a tankless system, is a type of upfeed system There is no 'static feed system'

The allowable stress ratings for lumber in the building codes are based primarily on (A) the length of the lumber (B) species (C) cross sectional area of the lumber (D) whether the lumber is used as a floor or ceiling joist

(B) species The allowable stress rating for lumber in the building codes are based primarily on species. The stress ratings are not dependent on the span, the area of the section, or the framing used

An economizer cycle (A) cools only as much chilled water as required by the demand load (B) uses outdoor air to cool a building (C) automatically reduces the amount of time the compressor runs (D) uses air and water to cool the condenser coils

(B) uses outdoor air to cool a building An economizer cycle introduces outdoor air when the ambient temperature is low enough to assist in cooling

Which of the following metals are cathodes? (Choose all that apply.) (A) Aluminum (B) Stainless steel (C) Copper (D) Gold (E) Zinc

(B), (C), (D) According to Olin's Construction Principles, Materials, and Methods, the electrolytic solution potentials in volts for metals are as follows: Magnesium: -1.73 Zinc: -1.00 Aluminum: -0.83 Cadmium: -0.82 Mild Steel: -0.58 Lead: -0.55 Tin: -0.49 Titanium: -0.30 Brass: -0.28 Copper: -0.20 Stainless Steel: -0.15 Silver: -0.15 Gold: -0.00 The closer the voltage potential is to 0.00 the more cathodic each metal is. The further from 0, the more anodic each metal is

Which of the following are common uses for laminated glass? (Choose the three that apply.) (A) Apartment windows (B) Car windshields (C) Prison windows (D) Skylights

(B), (C), (D) Laminated safety glass is best suited for the listed items: car windshields, prison windows, and skylights

Which of the following common building materials are considered non-combustible? (Choose all that apply.) (A) Aluminum (B) Brick (C) Concrete (D) CMU (E) Steel (F) Light wood framing (G) Heavy timber

(B), (C), (D), (E) Tests conducted according to ASTM E 136 and ASTM E 84 determine whether a 'substance' is classified as combustible or non-combustible. It is important to know whether common building materials are classified as combustible or not in order to determine the correct construction type Aluminum cannot pass the ASTM E 136, Vertical Tube Furnace at 750°C , test and so it is considered combustible Wood is combustible. This includes heavy timber (HT)

Which of the following are types of HVAC systems? (Choose the four that apply.) (A) Air-electric systems (B) Air-water systems (C) All-air systems (D) All-water systems (E) Electric systems (F) Water-electric systems

(B), (C), (D), (E) The four types of HVAC systems are: Air-water systems All-air systems All-water systems Electric systems

Water piping is most commonly made from which materials? (Choose the four that apply.) (A) Aluminum (B) Steel (C) Plastic (D) Brass (E) Lead (F) Copper

(B), (C), (D), (F) Water piping is most commonly made from copper, steel, plastic, and brass. Lead piping was very common until just after World War II when the effects of lead poisoning became known. Lead was used because of its unique ability to resist pinhole leaks, while being soft enough to form into shapes that deliver water efficiently Fun Fact Interesting enough, the word plumbing is derived from the Latin word for lead, "plumbum"

Select the true statements about galvanic action. (Choose the three that apply.) (A) The cathode will always corrode (B) The anode will always corrode (C) Cathodes with larger area create corrosion more quickly (D) Anodes with larger area create corrosion more quickly (E) A cathode is more noble than an anode (F) An anode is more noble than a cathode

(B), (C), (E) Galvanic corrosion occurs when two dissimilar metals come into contact in a wet or moist environment and conduct electricity. In this process, one metal is plated, the cathode, while the other corrodes, the anode. To avoid corrosion, only metals with similar voltage potential should be placed in contact with one another + The anode will always corrode + Cathodes with larger area create corrosion more quickly + A cathode is more noble than an anode - a noble metal is simply one that is superior in resistance to corrosion or oxidation

What symptoms commonly develop from exposure to VOCs? (Choose the three that apply.) (A) Diarrhea (B) Headaches (C) Nausea (D) Bleeding (E) Dehydration (F) Fatigue

(B), (C), (F) Key signs or symptoms associated with exposure to VOCs include: + conjunctival irritation + nose and throat discomfort + headache + allergic skin reaction + dyspnea + declines in serum cholinesterase levels + nausea + emesis + epistaxis + fatigue + dizziness

Which of these statements about mediation and arbitration are correct? (Choose the three that apply.) (A) Both mediation and arbitration or litigation are legally binding (B) Arbitration is more formal than mediation (C) The decisions for both mediation and arbitration cannot be appealed (D) An arbitrator can be either a single party or a panel (E) Litigation is typically more expensive than mediation or arbitration (F) A professional liability insurance policy will cover all the costs of litigation

(B), (D), (E) Mediation is not legally binding, while arbitration is Arbitration is more formal than mediation The decision for mediation can be appealed, while the decision for arbitration cannot An arbitrator can either be a single party or a panel Litigation is typically more expensive than mediation or arbitration A professional liability insurance policy will not cover all the costs of litigation

What characteristics are true of quartersawn wood? (Choose the three that apply.) (A) Has raised grain (B) Relatively even grain patterns (C) Shrinks, and swells less in thickness (D) Shrinks, and swells less in width (E) Doesn't utilize logs as effectively and thus results in more waste and higher cost (F) Has variety of noticeable grain patterns

(B), (D), (E) Quartersawn wood results in lumber that has a relatively even grain pattern, shrinks and swells less in width, more in thickness, and wears evenly with less raised grain and warping This type of cutting creates lumber that is more stable than the plainsawing method. However quartersawing doesn't cut logs in the most efficient manner so there is a lot of waste and the lumber is more expensive

Which of the following are considered passive HVAC solutions? (Choose the three that apply.) (A) Wind turbines (B) Geothermal (C) Radiant flooring (D) Solar heating (E) Natural ventilation

(B), (D), (E) Solar heating, geothermal, and natural ventilation are all types of passive HVAC. Wind turbines are a passive way to provide electricity to an HVAC system but aren't used to heat or cool a building. Radiant flooring may or may not be a passive solution to HVAC depending on how the radiant coils are heated. Without additional information, it is not possible to determine whether this answer is correct or not. On the exam, it is important to select the best (or most obvious) answers while trying not to make assumptions.

Which of the following contributors to indoor air quality can be controlled by the heating, ventilating, and air conditioning (HVAC) system? (Choose the four that apply.) (A) outgassing (B) air change effectiveness (C) volatile organic compound (VOC) content of building materials (D) carbon dioxide levels (E) humidity (F) mold growth

(B), (D), (E), (F) Mechanical systems cannot control outgassing or the level of VOC emissions from materials already incorporated into a building The HVAC system can control air change effectiveness and the levels of carbon dioxide found within the building. The HVAC system can also be relied on to keep humidity within human comfort range at acceptable levels to minimize mold growth

Galvanic action is most likely to occur between copper and which other materials? (Choose the three that apply.) (A) Galvanized Steel (B) Zinc (C) Acidic type wood (D) Steel (E) Bronze (F) Aluminum

(B), (D), (F) Copper is most likely to corrode with aluminum, iron/steel, stainless steel, and zinc. It should be noted that even though the potential for corrosion between other metals is lower, it is still possible According to Olin's Construction Principles, Materials, and Methods, the electrolytic solution potentials in volts for metals are as follows: Magnesium: -1.73 Zinc: -1.00 Aluminum: -0.83 Cadmium: -0.82 Mild Steel: -0.58 Lead: -0.55 Tin: -0.49 Titanium: -0.30 Brass: -0.28 Copper: -0.20 Stainless Steel: -0.15 Silver: -0.15 Gold: -0.00 The closer the voltage potential is to 0.00 the more cathodic each metal is. The further from 0, the more anodic each metal is

Galvanic action is most likely to occur between copper and which other materials? (Choose the three that apply.) (A) Galvanized Steel (B) Zinc (C) Acidic type wood (D) Steel (E) Bronze (F) Aluminum

(B), (D), (F) Copper is most likely to corrode with aluminum, iron/steel, stainless steel, and zinc. It should be noted that even though the potential for corrosion between other metals is lower, it is still possible Galvanized steel is comprised of a zinc coating

Which of the following are types of open specifications? (Choose the two that apply.) (A) Prescriptive (B) Reference (C) 'Approved Equal' (D) Base bid (E) Descriptive

(B), (E) Open specifications describe the results of the final construction assembly but leave it up to the contractor to select products which meet the requirements A descriptive specification gives detailed information on how a material or product is to be assembled, installed, and finished. A reference specification is a variation of a descriptive specification but uses the aid of reference standards, such as ASTM, to describe some of the requirements. Both of these are open specifications because they aren't specifying a specific manufacturer product or material Closed specifications specify manufacturer names and inform the contractor of the exact product or material to use Base bid and proprietary specifications are types of prescriptive (or closed) specifications.

What division would a steel W-shape beam be located in? (A) 03 (B) 04 (C) 05 (D) 06 (E) 07

(C) 05 Division 01 - General Requirements Division 02 - Existing Conditions Division 03 - Concrete Division 04 - Masonry Division 05 - Metals Division 06 - Wood, Plastics, & Composites Division 07 - Thermal & Moisture Protection Division 08 - Openings Division 09 - Finishes Division 10 - Specialties Division 11 - Equipment Division 12 - Furnishings Division 13 - Special Construction Division 14 - Conveying Equipment

What division is door hardware be located in? (A) 10 (B) 05 (C) 08 (D) 04

(C) 08 Division 01 - General Requirements Division 02 - Existing Conditions Division 03 - Concrete Division 04 - Masonry Division 05 - Metals Division 06 - Wood, Plastics, & Composites Division 07 - Thermal & Moisture Protection Division 08 - Openings Division 09 - Finishes Division 10 - Specialties Division 11 - Equipment Division 12 - Furnishings Division 13 - Special Construction Division 14 - Conveying Equipment

What is the approximate thickness of double-strength glass? (A) 1/32" (B) 3/32" (C) 1/8" (D) 1/4"

(C) 1/8" Single-strength glass is approximately 3/32″ thick Double-strength glass is approximately 1/8″ thick

An ordinary sprinkler heads activate in what temperature range? (A) 95° - 150° F (B) 115° - 150° F (C) 135° - 170° F (D) 175° - 250° F

(C) 135° - 170° F Ordinary sprinkler heads activate between 135° - 170° F and are color coded black with an orange or red glass bulb or fusible link. During a fire, the glass bulb or fusible link is heated and breaks. Once the bulb or link are no longer intact, the sprinkler head is free to open and water can flow. The water continues to flow until the water supply is shut off

There is concern about the workability of a concrete pour. There is also additional concern that the cured concrete will be exposed to freezing conditions and salt. Which kind of admixture is most appropriate to add to the concrete mix given these conditions? (A) Water-Reducing (B) Retarding (C) Air-entraining (D) Accelerating

(C) Air-entraining Air entraining agents would be best to use in this situation because they increase workability and help in conditions where frost and salt are a concern Air entrainment introduces microscopic air bubbles to a concrete mix. The entraining materials are restricted to plant or transit-mixed materials such as wood resin, sulfonated hydrocarbons, and fatty resinous acids

Which medium transmits sound the fastest? (A) Concrete (B) Air (C) Aluminum (D) Water

(C) Aluminum Sound transmission is related to density. The more dense something is the more easily it transmits sound

Which statement about an architect's responsibilities is correct? (A) An architect is responsible for continuous site inspections and safety conditions on the job site (B) An architects is responsible for methods and means of construction (C) An architect is responsible for failures to implement design changes in construction (D) An architect is responsible for construction time, cost overruns, and contractor failures in construction

(C) An architect is responsible for failures to implement design changes in construction Per A201, the contractor is responsible for safety, means of construction, construction cost, and the construction schedule

Which type of sprinkler system is filled with antifreeze to prevent freezing? (A) Deluge system (B) Dry pipe system (C) Antifreeze system (D) Wet pipe system

(C) Antifreeze system This type of system is (was) used when the area needing a sprinkler system is too small to accommodate a dry pipe system. (The NFPA outlawed them in 2013) A deluge system is when all sprinklers in a building discharge at the same time

What type of plywood is used for roof sheathing (A) A-B Exterior (B) B-C Exterior (C) C-D Exterior (D) A-D Exterior

(C) C-D Exterior The most common grade of plywood used for roof sheathing is C-D Exterior. sometimes referred to as CDX. The Engineered Wood Association classifies plywood veneers into six grades, as follows N = natural finish, free of defects A = smooth and paintable B = solid-surface veneer C = knotholes permitted to 1", limited splits D = knots/knotholes to 3" permitted, limited splits The first letter indicates the classification of the face ply (the highest grade side) of the plywood. (If both sides have the same classification, they are both referred to as faces.) The second letter indicates the classification of the back ply (the lower-grade side) of the material. If the plywood is marked exterior, it contains plies bonded with exterior glue

Control joints are also referred to as what? (A) Cold joints (B) Construction joints (C) Contraction joints (D) Guide joints

(C) Contraction joints Cold joints are used when a concrete slab is too large to be completed in a single concrete pour Construction joints are used where two separate placements of concrete meet and where reinforcement may be continuous Control joints (contraction joints) are tooled, sawed, or formed grooves in the concrete to create weak points where cracking can occur Isolation joints are used where adjacent areas of concrete may move in three directions and where the formation of cracks is to be avoided

Interior building mold is a result of what kind of conditions? (A) Hot (B) Dry (C) Damp (D) Cold

(C) Damp When mold spores drop on places where there is excessive moisture, such as where leakage may have occurred in roofs, pipes, walls, plant pots, or where there has been flooding, they will grow. Many building materials provide suitable nutrients that encourage mold growth. Wet cellulose materials, including paper and paper products, cardboard, ceiling tiles, wood, and wood products, are particularly conducive for the growth of some molds. Other materials such as dust, paints, wallpaper, insulation materials, drywall, carpet, fabric, and upholstery, commonly support mold growth

What is the movement of a beam from its original location when load is applied to it called? (A) Bracing (B) Moment (C) Deflection (D) Shearing

(C) Deflection According to the Dictionary of Architecture and Construction, deflection is: 1 - Any displacement in a body from its static position, or from an established direction or plane, as a result of forces acting on the body 2 - The deformation of a structural member as a result of loads acting on it

What type of water system utilizes multiple pumps that turn on/off as needed to deal with demand? (A) Static pressure head (B) Upfeed system (C) Direct upfeed pumping system (D) Downfeed system (E) Hydraulic system

(C) Direct upfeed pumping system Direct upfeed pumping systems (tankless systems) utilize multiple pumps that turn on/off as needed to deal with demand

Which of the following is NOT an egress component? (A) Exit (B) Exit Access (C) Exit Surcharge (D) Exit Discharge

(C) Exit Surcharge There is no such thing as an exit surcharge The egress components per the IBC in order from interior to exterior are 1 - Exit Access 2 - Exit 3 - Exit Discharge

Arches typically have ______ supports (A) Fixed (B) Flexible (C) Hinged (D) Magnetic

(C) Hinged Arches typically have 2 or 3 hinged supports. Fixed supports are only used on arches with short spans where expansion and contraction is minimal

How does the size of lumber compare from pre-manufacturing to post-manufacturing? (A) It is smaller in pre-manufacturing (B) The size is unaffected (C) It is smaller in post-manufacturing

(C) It is smaller in post-manufacturing During the manufacturing process, lumber is dried. The process of losing moisture causes the wood to shrink

Which drying method leaves lumber with the lowest moisture content? (A) Air drying (B) Evaporative drying (C) Kiln drying (D) Tunnel drying

(C) Kiln drying

Which mortar type is most suitable for use when masonry is subject to high lateral or compressive loads? (A) N (B) S (C) M (D) O

(C) M Type M mortar is high-strength mortar. It is suitable when masonry is subject to high lateral loads, compressive loads, severe frost, or masonry below grade. It should be specified for exterior applications at or below grade Type N or S is the best for exterior applications above grade and for interior load-bearing walls Type O is well suited for interior and protected exterior non-load-bearing partitions

What do albedo and conductivity affect? (A) Nanoclimate (B) Global climate (C) Microclimate (D) Macroclimate

(C) Microclimate Together, albedo and conductivity affect a site's microclimate. Albedo is the amount of radiation absorbed or reflected by a surface material. Conductivity is the rate of heat flow through a material. Surfaces on a site, such as grass, with high conductivity and low albedo moderate and stabilize the microclimate because extra heat is absorbed, stored, and then released when the temperature drops Surfaces with low conductivity and high albedo reflect heat making a microclimate hotter than what would be typical for its macroclimate

Where is the most appropriate place to locate a thermostat in a residential project? (A) On an exterior wall away from heat sources (B) On an exterior wall near a heat source (C) On an interior wall away from heat sources (D) On an interior wall near a heat source

(C) On an interior wall away from heat sources While the location of thermostats is typically specified by the mechanical engineer, it is important for an Architect to know generally where they should be placed in order to conduct adequate reviews of consultant drawings Typically thermostats should be located on interior walls away from any heat or cooling source that has the potential to give the thermostat an inaccurate snapshot of the general temperature within the space. This is true regardless of building use

Which of the following are suitable for drinking? (Choose all that apply.) (A) Greywater (B) Blackwater (C) Potable water (D) Greenwater

(C) Potable water Only potable water is suitable for drinking. Greywater is water that has been used but does not contain or has been in contact with human waste. Blackwater is water that contains or has the potential to contain human waste

Bids for a project come in over budget by $4,000. The contractor offers some ways to reduce the budget, which value engineering option would be most appropriate? (A) Remove the front entry canopy (B) Replace R-30 roof insulation with R-19 (C) Replace the tile with a more economic option (D) Remove the gravel fill under the basement slab

(C) Replace the tile with a more economic option In cases where value engineering is used the architect should always recommend choices that allow for the overall design intent to remain while also retaining the building's integrity. Lowering the R-value of insulation below recommended levels may save on upfront costs but will create higher heating and cooling bills. The front entry canopy is likely a major design component and removing it would detract from the overall design intent. If gravel fill was specified under a concrete slab, there is concern about hydrostatic pressure and it should remain to prevent flooding in the basement. The option that should be considered is to replace the tile with a more economic option. Architects tend to prefer high-end materials. In a pinch, a quick way to save money on a project is to revisit specified finishes

What unit measures sound absorbency? (A) Hertz (B) Decibel (C) Sabin (D) Sone

(C) Sabin Sabin is the unit of measurement for sound absorbency

Which material has the highest allowable compressive strength? (A) Wood (B) Brick (C) Steel (D) Concrete

(C) Steel Allowable compressive strength: Brick: 250 psi Wood: 1,100 psi Concrete: 1,350 psi Steel: 22,000 psi

Which material has the highest allowable tensile strength? (A) Wood (B) Brick (C) Steel (D) Concrete

(C) Steel Average tensile strength Wood: 700 psi Brick: 0 psi Steel: 22,000 psi Concrete: 0 psi

Which of the following has the highest density and modulus of elasticity? (A) Brick masonry (B) Concrete (C) Steel (D) Wood

(C) Steel The higher a material's modulus of elasticity, the greater ability it has to bend Modulus of Elasticity: Wood: 1,200,000 psi Brick: 1,200,000 psi Concrete: 3,150,000 psi Steel: 29,000,000 psi

How do wet-bulb and dry-bulb temperatures differ at 100% relative humidity? (A) The wet-bulb temperature is greater than the dry-bulb temperature (B) The dry-bulb temperature is greater than the wet-bulb temperature (C) The dry-bulb temperature and wet bulb temperature are the same (D) Not enough information is given in this question to give an answer

(C) The dry-bulb temperature and wet bulb temperature are the same At 100% relative humidity the dry bulb temperature and wet bulb temperature are the same. 100% relative humidity means that the air is completely saturated with water vapor. In this case the "dry" bulb would be effectively the same as the "wet" bulb

In a W12x210 column, what does the 210 represent? (A) The 210 is simply an identification number (B) The allowable pounds per linear foot (C) The weight of the steel per linear foot (D) The flange width in millimeters

(C) The weight of the steel per linear foot In a W12x210 steel I-beam, the 210 represents the weight of the steel per linear foot. The 12 represents the approximate depth of the web, and may vary a few inches in either direction

Which statement about unit prices is true? (A) Unit prices for added work must be the same as for the deleted work (B) Unit prices are the same as alternates (C) Unit prices are stated in the bid (D) Unit prices are used whenever the Contractor submits an application for payment

(C) Unit prices are stated in the bid Unit prices supply a price per unit for materials and/or services. They offer the Architect and Owner a comparable means of measurement to use in comparing bids

Upfeed water supply systems are typically used at max building heights of __________ (A) between 20 and 40 feet (B) between 30 and 50 feet (C) between 40 and 60 feet (D) between 50 and 70 feet

(C) between 40 and 60 feet Upfeed water supply systems are typically used at max building heights of between 40 and 60 feet In an upfeed system, the water within the building comes directly from the water main. As the water travels up through the building it loses pressure the higher it goes Since fixtures have psi requirements in order to operate, the max range for an upfeed system is between 40 and 60 feet

Which type of fire extinguisher should be provided in an electrical room? (A) class A (B) class B (C) class C (D) class D

(C) class C Class A = regular combustible materials Class B = flammable gases Class C = electrical fires Class D = combustible metals

What type of ceiling is best to avoid when sound control is important? (A) flat (B) sloped (C) concave (D) convex

(C) concave It is best to avoid concave or dome-like ceilings when sound control is important. In a room with a concave ceiling, all of the sound is reflected back into the space. The distance and location the sound reflects from varies. This variation creates poor sound quality

What is the building code requirement for pairs of exit doors with astragals? (A) weather stripping (B) door stop (C) coordinator (D) flush bolts

(C) coordinator A coordinator prevents the door leaf with the astragal from closing before the other leaf, so the pair of doors seals properly

The direct wind pressure on a vertical surface is (A) directly proportional to the wind velocity (B) inversely proportional to the wind velocity (C) directly proportional to the square of the wind velocity (D) not related to the wind velocity

(C) directly proportional to the square of the wind velocity p = 0.00256v^2 where v is given in mph and p is in lbs/ft2

According to the US National CAD standard system of organizing construction documents, what type of drawings would one expect to find on sheet A-302? (A) wall sections (B) fourth-floor plan (C) exterior elevations (D) reflected ceiling plans

(C) exterior elevations

An architect is designing a decorative wood grille wallcovering in a building lobby. The grille is attached to a gypsum board partition with metal clips. Which item should be checked first? (A) spacing and size of the screws holding the clips to the partition (B) possibility of splintering dues to the way the exposed surfaces are milled (C) flame-spread rating (D) method of cleaning the grille based on design and finish techniques used

(C) flame-spread rating Fire safety is the most important

In a high-rise building over 500 feet tall, what kind of elevator is most-likely to be specified? (A) hydraulic elevator (B) geared traction elevator (C) gearless traction elevator (D) MRL elevator

(C) gearless traction elevator In a high-rise building it makes the most sense to use a gearless traction elevator. Gearless traction elevators can travel up to 2,000 feet. The next highest travel distance is geared traction which can travel up to 250 feet

The overall thickness of a one-way reinforced concrete slab is often determined based on American Concrete Institute (ACI) 318 provisions for minimum thickness. According to these provisions, what should be the overall thickness for a cantilevered concrete slab if the slab span is 11 ft? (A) 5.0" (B) 5.5" (C) 7.0" (D) 13"

(D) 13" According to ACI 318, the minimum overall thickness of a cantilevered slab os span L is L/10. The minimum thickness, h, then is h = = L/10 = (11 ft)(12 in/ft) / 10 = 13.2"

In preparing the contract documents, the owner asks what the architect's responsibility is if the architect finds a safety violation during a job site visit. The architect should (A) do nothing (B) prepare a change order to correct all safety violation (C) notify the owner and follow up in writing (D) write to the contractor to suggest how to correct the problem

(C) notify the owner and follow up in writing Although the architect is not responsible for safety issues on the job site, the matter is to be brought to the attention of the owner without a suggestion for correction. The architect also follows up wit ha notice in writing. It is the responsibility of the contractor to rectify safety problems as described in AIA A201

Plans, sections, and elevations are examples of (A) isometric drawings (B) axonometric drawings (C) orthographic drawings (D) oblique drawings

(C) orthographic drawings

Aluminum windows are specified for installation in a masonry wall. Which material is the least desirable choice for flashing at the head of the window? (A) aluminum (B) copper (C) stainless steel (passive) (D) lead

(C) stainless steel (passive) Stainless steel would be the least desirable choice for flashing material when used near an aluminum window assembly because of the metals listed, it is the furthest from aluminum on the galvanic action table. The more dissimilar the materials, the more current that will flow between the two and speed corrosion. This process is called electrolysis The best choice would be to use aluminum flashing and aluminum fasteners and accessories. However, aluminum can also react with alkaline materials, so it must be protected from mortar and concrete. Nonreactive materials such as plastics are also an option, but they should be carefully researched, as plastics tend to degrade more quickly than metals. Any dissimilar materials in proximity to one another must always be carefully separated with a layer of nonreactive material such as runner or neoprene

Which types of plastic pipes are flexible? (Choose the two that apply.) (A) CPVC (B) ABS (C) PP (D) PE (E) PEX-AL-PEX

(D), (E) The following plastic pipes are flexible PEX-AL-PEX, PE The following plastic pipes are rigid ABS, CPVC, PP, PVC HDPE may be rigid or flexible

Which of the items listed below are ingredients in portland cement? (A) Aggregate (B) Admixtures (C) Limestone (D) Clay (E) Iron ore (F) Gypsum

(C), (D), (E), (F) Portland cement is composed of limestone, clay, iron ore, and gypsum. Portland cement should not be confused with concrete which contains aggregate and may contain admixtures

An Architect is drawing a CMU wall on the plans. What nominal thickness is most appropriate for the Architect to draw the wall at? (A) 3 inches (B) 5 inches (C) 8 inches (D) 11 inches (E) 12 inches

(C), (E) A Concrete Masonry Unit (CMU) wall in plan can have a nominal thickness of 4, 6, 8, 10, or 12 inches

In what 2 ways can the health risks of asbestos found in building renovation be avoided? (A) Heat-drying (B) Adequate cross-ventilation (C) Abatement (D) Vaccuming (E) Encapsulation

(C), (E) If asbestos is discovered during construction a specialist should be hired to assess the situation. The specialist will most likely recommend abatement (removing) the asbestos or encapsulating (sealing) it. Asbestos is only problematic when it becomes airborne

Which of the following structures require some complex wind calculations and possibly some wind tunnel testing? (Choose the three that apply.) (A) commercial building with a total height of 50 ft (B) residential building with a height of 200 ft (C) office building with a height of 450 ft (D) building with a height-to-width ratio of 3 (E) building with a height-to-width ratio of 7 (F) extra-long span suspension bridge

(C), (E), (F) Generally, complex wind calculations and wind tunnel testing are required for buildings with heights exceeding 400', for buildings subject to dynamic effects, those sensitive to wind vibration, and for buildings with a height-to-width ratio of 5 or more. Wind tunnel testing is often carried out on reduced-scale models of long-span suspension bridges

Of the items listed below which are types of mixing box systems? (Choose the three that apply.) (A) Single-duct system (B) Static air volume system (C) Dual-duct system (D) Coil trap system (E) Variable air volume system (F) Terminal reheat system

(C), (E), (F) The three mixing box systems are: Dual-duct system - Combines hot and cool air from two separate ducts to a desired temperature. Terminal reheat system - Uses hot water coils to reheat the air as needed. Variable air volume (VAV) system - Receives heated or cooled air from a main supply and distributes it to a location as needed.

Which of the following metals are most resistant to oxidization? (A) Cast iron (B) Copper (C) Gold (D) Steel (E) Aluminum (F) Silver

(C), (F) Oxidization is a process that occurs in non-precious (or noble) metals when moisture causes the metal to react with oxygen molecules Of the listed metals, gold and silver are the most noble and least likely to oxidize or rust Oxidization and corrosion are very similar. Corrosion usually refers to a loss of material Oxidation is common with iron. Oxygen and iron combine to form iron oxide or rust

Radon testing is conducted in the basement of an elementary school that is under study for a renovation and addition project. The results show a concentration of 3 pCi/L. Determine an appropriate course of action (A) Demolish the existing slab, and install a new 4" slab on a vapor barrier placed on top of a 4" base course of gravel (B) Seal any cracks in the foundation walls and floor slab, and ventilate the basement to the exterior (C) Install a membrane on the floor and ventilate beneath it (D) No action is required at this time, but the site should be monitored

(D) Radon is a colorless, odorless gas that has been shown to cause lung cancer. It is found in the earth. Testing is a relatively simple and inexpensive process. The EPA has determined that no action is required if the level of radon is less than 4 pCi/L. However, because this site shows an elevated radon reading, it should be monitored with periodic testing. Should an addition be constructed, steps should be taken to reduce radon levels by providing proper ventilation of spaces in direct contact with the earth Appropriate remedial actions for concentrations over 4 pCi/L include sealing any cracks in the foundation walls or floor slab and ventilating or depressurizing the basement or crawlspace area. The EPA recommends that new residences be built with a radon-resistant techniques noted at epa.gov

In what CSI MasterFormat division would CMU be located in? (A) 41 (B) 05 (C) 23 (D) 04

(D) 04 Division 01 - General Requirements Division 02 - Existing Conditions Division 03 - Concrete Division 04 - Masonry Division 05 - Metals Division 06 - Wood, Plastics, & Composites Division 07 - Thermal & Moisture Protection Division 08 - Openings Division 09 - Finishes Division 10 - Specialties Division 11 - Equipment Division 12 - Furnishings Division 13 - Special Construction Division 14 - Conveying Equipment

What division would a skylight be located in? (A) 26 (B) 05 (C) 17 (D) 08

(D) 08 Skylights are listed under Division 08 - Openings (under "Roof Windows and Skylights")

In order to connect two levels whose difference in elevation is two feet, a handicapped ramp must be at least ________ (A) 10 feet long (B) 12 feet long (C) 24 feet long (D) 34 feet long

(D) 34 feet long The maximum rise of an ADA ramp is 1:12. A 24″ rise would require a 24′-0″ long ramp with 5′-0″ top and bottom landings. For a total of 34′-0″

The surface of a water tank is 100' above the lowest plumbing fixture in a building. Ignore friction loss. The water pressure available at the lowest fixture is nearly (A) 23 psi (B) 31 psi (C) 36 psi (D) 43 psi

(D) 43 psi 0.433 psi is required to lift water a vertical distance of 1'-0". Viewed another way, 0.433 psi of pressure is developed for every 1' of height. This is called the static head pressure = = (0.433 psi/ft) (100 ft) = 43.3 psi

For three story residential construction, what is the minimum spacing at which anchor bolts are permitted to be placed to secure a sill plate? (A) 12" o.c. (B) 16" o.c. (C) 24" o.c. (D) 48" o.c. (E) At the two ends

(D) 48" o.c. The IBC permits anchor bolts to be spaced at 6′-0″. However there is an exception which states that for buildings over two stories in height, the maximum anchor bolt spacing shall be 4′-0″ or 48". It is standard practice to use 4′-0" for anchor bolt spacing

A "design day" is colder than _____ of days. (A) 50% (B) 90% (C) 95% (D) 98%

(D) 98% A design day is colder than 98% of days. Design days are used to calculate the required size of heating equipment in a building. If the heating in a building is able to keep a building warm on a day colder than 98% of days than it is sufficiently sized for 98% of conditions

A building's construction type would not affect the design of which one the below items? (A) Building materials (B) Floor area (C) Building height (D) Accessibility requirements

(D) Accessibility requirements Floor area, building height, and building materials are all affected by a building's construction type (per Table 503 in the IBC)

What is a pro forma? (A) Providing free service in the interest of a public or deserving cause (B) A fixed-price set by an Owner for a construction project (C) A detailed cost description that outlines each item to be included in a project (D) Assumed, forecasted, or informal information presented in advance of the actual or formal information

(D) Assumed, forecasted, or informal information presented in advance of the actual or formal information A pro forma is assumed, forecasted, or informal information presented in advance of the actual or formal information. The common objective of a pro forma document is to give a fair idea of the cash outlay for an anticipated occurrence. Pro forma financial statements give an idea of how the actual statement will look if the underlying assumptions hold true. Latin for, according to form or for form's sake

What phase of a project takes up the smallest percentage of the architect's fee? (A) Schematic design (B) Design development (C) Construction documentation (D) Bidding & negotiation (E) Construction administration

(D) Bidding & negotiation Traditionally, the bidding & negotiation phase takes up the smallest chunk of time for the Architect. This phase accounts for around 5% of the overall time spent on a project. The Architect's role during the phase is to issue documents to bidders, respond to RFIs, and issue addenda

Fluorescent lighting tends to give off what color light? (A) White (B) Green (C) Yellow (D) Blue

(D) Blue Fluorescent lighting is known to give off blueish light. When this blueish light hits surfaces of cool colors it tends to brighten them. When it hits surfaces of warm colors, it tends to dull them

An architect wants to increase the expected lighting level of a room. Which of the following steps could accomplish this? (A) Change to a lamp type with a lower efficiency (B) Suggest to the owner that the lamps be replaced infrequently (C) Use finishes with a lower light reflectance value (D) Change to luminaires with a higher coefficient of utilization

(D) Change to luminaires with a higher coefficient of utilization Luminaires with a higher coefficient of utilization (CU) allow more light from the lamps to reach the desired surfaces Lamps with a higher efficiency should be selected, although the selection of this type of fixture would have to be balanced against the change in color temperature Changing lamps often would help maintain the initial footcandle level Room finishes with high light reflectance values can make a significant increase in the total light level in a room

Preconstruction conferences are conducted in order to: (A) Determine whether a performance bond and a payment bond are required (B) Discuss changes to completed work (C) Review bonds and affidavits (D) Clarify responsibilities and operating procedures

(D) Clarify responsibilities and operating procedures The correct answer is clarify responsibilities and operating procedures Bonds are an important topic but should be discussed with legal council A preconstruction meeting would mean no work has been completed so changes to completed work cannot be discussed

Which material would provide the most passive heat during the winter? (A) Wood near south facing windows (B) Concrete near north facing windows (C) Carpet and wall coverings throughout the building (D) Concrete near south facing windows

(D) Concrete near south facing windows Concrete is a thermal mass. This means that is has the ability to absorb and store heat energy Using concrete on the south of a building where sunlight can hit it allows it to be heated throughout the day. When the sun goes down, the concrete radiates the heat it gained throughout the day, thereby passively heating the space

What division is steel concrete reinforcement be located in? (A) Division 01 (B) Division 03 (C) Division 04 (D) Division 05 (E) Division 06 (F) Division 07

(D) Division 03 Although steel is a metal, steel reinforcing is a component used in concrete and is therefore listed under division 03 -concrete. Specifically, reinforcing steel is found under section 03 21 00 Division 01 - General Requirements Division 02 - Existing Conditions Division 03 - Concrete Division 04 - Masonry Division 05 - Metals Division 06 - Wood, Plastics, & Composites Division 07 - Thermal & Moisture Protection Division 08 - Openings Division 09 - Finishes Division 10 - Specialties Division 11 - Equipment Division 12 - Furnishings Division 13 - Special Construction Division 14 - Conveying Equipment

What causes portland cement to harden? (A) Exposure to heat (B) Exposure to open air (C) Exposure to sub-freezing temperatures (D) Exposure to water

(D) Exposure to water Portland cement is hardened when water is added through a process called hydration. Hydration is a chemical process where water bonds together with other molecules. In the creation of concrete, this process is mostly exothermic meaning that the reaction generates heat

What material is associated with SHGC? (A) Insulation (B) Membrane roofing (C) Wood (D) Glass

(D) Glass SHGC is the solar heat gain coefficient. SHGC is the percent of solar energy incident on glass that is transferred indoors both directly and indirectly through the glass

Which of the following site elements/materials would provide the lowest albedo? (A) Lake (B) Concrete sidewalk (C) Glass (D) Grass

(D) Grass Albedo is the amount of light that is reflected off of a material/surface back into space If you've ever gone outside after it snowed you know how bright it can be. Fresh snow has an albedo between 0.8 and 0.9. Grass has an albedo of 0.25. The higher the albedo the more reflective something is Of the listed items, grass has the lowest albedo

A decorative concrete floor has been poured, but the structural engineer has expressed concern about the bearing capacity of the slab. What test should be done to test the strength of the concrete without damaging the floor? (A) K-slump test (B) Cylinder test (C) Core cylinder test (D) Impact hammer test (E) Kelly ball test

(D) Impact hammer test An impact hammer test should be conducted. In this test, a spring-loaded plunger is hammered against the concrete surface. The amount of rebound approximates the strength of the concrete

Why is steel used to reinforce concrete? (A) Improve bond strength (B) Improve durability (C) Improve rigidity (D) Improve tensile strength (E) Improve compressive strength

(D) Improve tensile strength Steel is used to reinforce concrete because concrete is strong is compression but not tension. Including steel rebar in concrete improves the concrete's strength in tension. Both have similar thermal rates of expansion

Which type of joints are used where adjacent areas of concrete may move in three directions and where the formation of cracks is to be avoided? (A) Cold (B) Construction (C) Contraction (D) Isolation

(D) Isolation Cold joints are used when a concrete slab is too large to be completed in a single concrete pour. Construction joints are used where two separate placements of concrete meet and where reinforcement may be continuous. Control joints (contraction joints) are tooled, sawed, or formed grooves in the concrete to create weak points where cracking can occur. Isolation joints are used where adjacent areas of concrete may move in three directions and where the formation of cracks is to be avoided

Which grade of copper pipe is commonly used for underground supply water pipes? (A) A (B) B (C) C (D) K (E) L (F) M (G) N

(D) K Copper pipe is classified in three grades: K, L, and M (arranged from thickest to thinnest walls) Grade K copper pipe is commonly used for underground supply water pipes Grade L copper pipe is commonly used for plumbing systems within a building Grade M copper pipe is commonly used in drainage piping, branch supply lines, chilled water systems, and exposed lines in heating systems

One of the jobs of a zoning ordinance is to promote social vitality. What zoning strategy would best promote social vitality? (A) Building height regulations (B) Cluster development (C) Distinct districts (D) Mixed-use & overlay zoning

(D) Mixed-use & overlay zoning It has been proven that a diversity of uses helps promote social vitality. Mixed-use and/or overlay zoning is the best zoning strategy to create this

How does a building's period cause it to react during an earthquake? (A) A lower building period decreases its response to ground shaking (B) A building's period does not affect its response to ground shaking (C) A lower building period increases a building's response to ground shaking (D) Not enough information is given in the question above to come to a definitive conclusion

(D) Not enough information is given in the question above to come to a definitive conclusion The most damage is caused to a building when the building's natural period matches the soil's period during an earthquake. This matching of both periods creates resonance. Resonance is the increase in acceleration of a vibrating object due to a harmony of periods FEMA 454 defines resonance as: "When a vibrating or swinging object is given further pushes that are also at its natural period, its vibrations increase dramatically in response to even rather small pushes and, in fact, its accelerations may increase as much as four or five times" Below are general ideas of the periods for soil types and building heights: Clay: long period Sand: intermediate period Rock: short period High-rise: long period Mid-rise: intermediate period Low-rise: short period

Laminated glass is made by sandwiching two pieces of glass together with a transparent interlayer of what? (A) MPa (B) ELO (C) Epoxy (D) PVB

(D) PVB Laminated glass is made by sandwiching two pieces of glass together with a transparent interlayer of polyvinyl butyral (PVB) or resin. Once the PVB is sandwiched between the glass, heat and pressure are applied to fuse the three into a single unit

Which type of fire detection device detects the presence of smoke? (A) Ionization detector (B) Gas-sensing detector (C) Particulate detector (D) Photoelectric detector

(D) Photoelectric detector A photoelectric detector emits a non-visible light beam that detects smoke when the particles of smoke cause the light beam to scatter Ionization detectors detect combustion-ionized particles not smoke A gas-sensing system detects combustion gases in the air A particulate detector is not a type of fire prevention system

Depending on the local jurisdiction, which of type of piping may or may not be used? (A) Steel (B) Copper (C) Lead (D) Plastic

(D) Plastic Depending on the jurisdiction plastic piping may or may not permitted Lead is not the correct answer because it is not permitted in any circumstance to transmit water according to the 1986 Safe Drinking Water Act

________________ is the method by which heat is transferred between two objects not in contact and not shielded from each other. (A) Conduction (B) Convection (C) Induction (D) Radiation

(D) Radiation Radiation is the method by which heat is transferred between two objects not in contact and not shielded from each other

Which of the following would qualify as fine aggregate? (A) Crushed stone (B) Gravel (C) Portland cement (D) Sand

(D) Sand Sand would quality as fine aggregate. Fine aggregate is typically anything smaller than 4.75 mm, per a sieve test Portland cement isn't an aggregate it's a binding agent

What causes efflorescence? (A) Environmental wear and tear (B) Mortar experiencing a chemical reaction with the brick (C) Oxidation over long periods of time (D) Soluble salts leaching out of the mortar and/or brick

(D) Soluble salts leaching out of the mortar and/or brick Efflorescence, is the migration of a salt to the surface of a porous material (commonly brick or mortar), where it forms a white coating on the surface

What is the system for measuring perceived loudness? (A) Decibel (B) Halon (C) Sabin (D) Sone

(D) Sone Sones are a unit of subjective or perceived loudness Decibels are an objective measurement of sound Sabin is a unit of sound absorption Halon is a gas used in fire supression

What is a British thermal unit (Btu)? (A) The amount of heat required to raise the temperature of one cubic foot of water by 1°F (B) The amount of heat required to raise the temperature of one cubic meter of water by 1°C (C) The amount of heat required to raise the temperature of one kilogram of water by 1°C (D) The amount of heat required to raise the temperature of one pound of water by 1°F

(D) The amount of heat required to raise the temperature of one pound of water by 1°F Fun Fact: Despite its name, this non-metric unit is used more often in the US, Canada, and Caribbean than in Britain (or rest of the world) where calorie is preferred

What does a K-value denote? (A) Density of a material compared to the standard density of that material type (B) Elasticity of a material (C) Tensile strength of a material (D) Thermal conductivity of a material

(D) Thermal conductivity of a material K-value is the thermal conductivity of a material

What does U-factor or U-value measure? (A) Sound absorption of a particular building element or material (B) Sound conductivity of a particular building element or material (C) Thermal absorption of a particular building element or material (D) Thermal conductivity of a particular building element or material

(D) Thermal conductivity of a particular building element or material The U-value measures the thermal conductivity of a particular building element or material. The U-value is the inverse of the R-value: R = 1/U

In order to obtain the best price for a small project, the Owner and the Architect should solicit bids from ____________. (A) Two of the best known contractors (B) The three largest general contractors (C) The three smallest general contractors (D) Three to five of the most competitive contractors

(D) Three to five of the most competitive contractors Regardless of size, the way to get the best price on a project to create competition among the Contractors who want the work

A massive brick wall behind glazing on the south side of a house would act as a (A) direct gain system (B) greenhouse design (C) passive system with active assist (D) Trombe wall

(D) Trombe wall A Trombe wall is a type of thermal storage wall that is placed directly behind glass on the south side of a building to store solar energy during the day for release at night A direct gain system uses various types of massive materials (masonry, concrete, or even gypsum board) in different locations inside a building to store heat A greenhouse uses a south-facing room enclosed with extensive glass, not masonry A passive system with active assist include mechanical devices in addition to thermal mass

An incandescent bulb has a _______ filament. (A) Copper (B) Lead (C) Steel (D) Tungsten

(D) Tungsten An incandescent light bulb has a tungsten filament. When an electric current is applied to tungsten it causes it to heat up and glow

According to the 2012 IBC, which construction type is heavy timber? (A) Type I (B) Type II (C) Type III (D) Type IV (E) Type V

(D) Type IV

A Siamese fitting is a ________ hose attachment at the base of a building which allows the fire department to connect a fire hydrant through a pumper truck to provide or augment water flow to a standpipe. (A) J-shaped (B) P-shaped (C) X-shaped (D) Y-shaped

(D) Y-shaped A Siamese connection or splitter in fire protection engineering is a pipe fitting that allows two or more fire hoses to be connected to a single standpipe riser at the same general location

Which type of pipe is typically used for sanitary lines in nonresidential buildings? (A) PVC (B) copper (C) ABS (D) cast iron

(D) cast iron Cast iron pipe is noncorrosive in most soils and resists abrasion from waste materials that may be drained through the pipe as well as from rock and soil on the outside of the pipe in underground installations. It is quick to install, readily available, economical, and muffles sound PVC is often used for cold water supply lines. It is another option for sanitary lines. It cannot be used for exterior applications or where noise reduction is a consideration Copper pipe is generally used for supply lines ABS is primarily used for drainage lines in residential buildings

Devices that hold reinforcing steel in position and prevent the rebar from slipping out of place as concrete is placed are called (A) props (B) lifts (C) shims (D) chairs

(D) chairs Chairs are small wire supports that help to keep the rebar a specified distance from the outside of the concrete and help ensure adequate

Which of the following pedestrian walk materials provides the best positive grade-level drainage away from a building? (A) asphaltic concrete (B) brick pavers (C) cobblestones (D) concrete

(D) concrete A safe pedestrian walk should not have a slope exceeding 1/4"/ft perpendicular to the direction of travel. This allows for drainage without creating a dangerous cross slope. Of the materials listed, concrete could be finished to provide a continuous, uniform slope for drainage in conjunction with a smooth walking surface Asphalt could be used, but it is more difficult to smooth uniformly at such a low slope. Minor dips and surface irregularities in the asphalt might cause ponding of water against or towards the building Both brick pavers and cobblestone would allow water to seep into the joints near the building

A reference to 3000 psi concrete alludes to the concrete's (A) ultimate strength in psi (B) strength in tension (C) flexural strength without reinforcing (D) design strength in psi after curing for 28 days

(D) design strength in psi after curing for 28 days The design strength of the final mix of concrete is specified by the compressive strength of the concrete in lbs/in2 after it has cured for 28 days. Common strengths are 3,000 and 4,000 psi, although high-strength concrete is available up to 22,000 psi

What structural data must be included in both structural and architectural drawings? (A) footing details (B) rebar layouts (C) pier reinforcing schedules (D) elevations for tops of beams and structural walls

(D) elevations for tops of beams and structural walls (A), (B), and (C) are all found only in the structural drawings

What is the most important fire-resistance property of a concrete masonry unit (CMU) partition? (A) overall width (B) density (C) joint reinforcement (D) equivalent thickness

(D) equivalent thickness Concrete masonry partitions are usually hollow, so the actual thickness of the solid material, not the actual overall width, is used to rate the fire resistance of the unit

What is static head measured in? (A) area of air (B) volume of water (C) volume of air (D) inches of water

(D) inches of water The static head, sometimes referred to as the pressure head, is used to denote the static pressure in a pipe, channel, or duct flow. It is measured in inches of water

Which of the following items should be addressed in the supplementary conditions? (A) bidding instructions (B) demolition requirements (C) administrative procedures (D) legal requirements of the jurisdiction in which the project is located

(D) legal requirements of the jurisdiction in which the project is located The supplementary conditions should include information about basic legal rights and responsibilities that may vary from one project to another. This includes insurance, indemnification, liquidated damages, legal requirements of the jurisdiction that may differ from the standard language in the general conditions, and fiduciary obligations. Instructions for preparing the supplementary conditions are available inf AIA A503, Guide for Supplementary Conditions Administrative procedures are properly addressed in the specifications in Division 01, General Requirements Demolition Requirements are addressed in Division 02, Existing Conditions

An architect is consulting with an electrical engineer for a reflected ceiling plan. Which information must be given to the electrical engineer? (A) grille and register location (B) sprinkler head location (C) luminary location (D) lighting and power outlet circuiting

(D) lighting and power outlet circuiting The exact circuiting of lighting and power outlets does not have to be shown on the architect's reflected ceiling plan. Rather, electrical drawings must contain this information. In some cases, such as an office tenant improvement project, the architect shows the location of switches on the reflected ceiling plan if the electrical construction work will be subcontracted out as design-build work and no electrical engineer is consulting on the project

In what grades is copper pipe classified? (Choose the three that apply.) (A) A (B) B (C) C (D) K (E) L (F) M (G) N

(D), (E), (F) Copper pipe is classified in three grades: K, L, and M (arranged from thickest to thinnest walls) Grade K copper pipe is commonly used for underground supply water pipes Grade L copper pipe is commonly used for plumbing systems within a building Grade M copper pipe is commonly used in drainage piping, branch supply lines, chilled water systems, and exposed lines in heating systems

What division would wood shingle siding be located in? (A) 06 (B) 03 (C) 12 (D) 05 (E) 07

(E) 07 Although wood shakes are wood they are not listed in Division 06 under woods, plastics, and composites. Wood shake shingles are part of the building envelope and serve a critical role in keeping the elements out. Wood shakes are listed under Division 07 thermal and moisture protection. Specifically wood shingles and shakes are found under 07 31 29

Which grade of copper pipe is most commonly used for plumbing within a building? (A) A (B) B (C) C (D) K (E) L (F) M (G) N

(E) L Copper pipe is classified in three grades: K, L, and M (arranged from thickest to thinnest walls) Grade K copper pipe is commonly used for underground supply water pipes Grade L copper pipe is commonly used for plumbing systems within a building Grade M copper pipe is commonly used in drainage piping, branch supply lines, chilled water systems, and exposed lines in heating systems

On the side of a PVC pipe, what would the marking "NSF-pw" represent? (A) Not safe for potable water (B) National science foundation plastic white (C) Non-standard facilities plastic white (D) National science foundation plastic weldable (E) National sanitation foundation potable water

(E) National sanitation foundation potable water This marking identifies potable water that passes through this pipe has been deemed safe for drinking by the NSF. This marking exists because some plastic pipes leech chemicals into water which make it unsafe to drink

A single-family residential building constructed entirely of unprotected wood framing would be which construction type? (A) Type I (B) Type II (C) Type III (D) Type IV (E) Type V

(E) Type V Type I is made entirely of noncombustible materials such as concrete or stone Type II is made of noncombustible materials with the exception of the roof covering material Type III has masonry-bearing walls but floors, structural framework, and roof made of wood or other combustible materials Type IV is heavy timber This building is most likely Type V construction which is entirely combustible wood framed construction

Which grade of copper pipe is most commonly used for branch supply lines? (A) A (B) B (C) C (D) K (E) L (F) M (G) N

(F) M Copper pipe is classified in three grades: K, L, and M (arranged from thickest to thinnest walls) Grade K copper pipe is commonly used for underground supply water pipes Grade L copper pipe is commonly used for plumbing systems within a building Grade M copper pipe is commonly used in drainage piping, branch supply lines, chilled water systems, and exposed lines in heating systems

Reorganize the following list of drawing types into the correct order to assemble a set of typical construction drawings + architectural drawings + mechanical/electrical/plumbing drawings + civil engineering drawings + title sheet + structural drawings

+ title sheet + civil engineering drawings + architectural drawings + structural drawings + mechanical/electrical/plumbing drawings

A slope starts at elevation +512.61' and slopes up at 4% for 21'-9" feet of run. At the top of the slope, what is the elevation?

+513.48′ Set up a proportion to solve for the total rise 4/100 = x/21.75′ x = .87 Elevation = 512.61' + .87' = 513.48′

What is the maximum unsupported height for a solid brick wall 8" thick?

160" or 13'-4" The nominal thickness, t, of the wall is used in the calculation, so the maximum height is (8")(20) = 160" Even though single-wythe walls are not reinforced vertically, they must have a minimum amount of horizontal reinforcement, just as multi-wythe walls must. The most commonly used reinforcements are prefabricated assemblies consisting of minimum nine-gauge steel laid every 16"

A water tank is located on the top of a building. The Architect needs to place a shower near the top of the building. If the shower needs 12 psi, how much lower from the water tank should the shower be located?

28 ft Elevation changes can add or subtract water pressure in a water system. Each foot of elevation change is equal to 0.433 psi of water pressure Pressure Loss = = (12 ft) / (0.433 psi/ft) = 27.713 ft

A design for a rest station at an amusement park includes a group of six family restrooms, each containing a toilet, lavatory, and changing table. At least ________ of these restrooms must be accessible. (Fill in the blank.)

3 Unisex toilet rooms are required by the IBC where there are more than six separate-sex water closets required, and half of those provided (at least one) must be accessible. The fixtures provided in these rooms may count toward the total fixture requirement. Single-user toilet rooms are preferred by many for privacy and convenience. They are particularly useful in areas where there are lots of children because they make it possible for a parent to accompany a child of the opposite sec to the restroom

An architect is assisting with the renovation of a 1780 farmhouse in Virginia. The total floor area of the farmhouse is approximately 1,500 ft2. The owner requests a central air conditioning system. The cooling capacity required, rounded to the nearest ton, will be ________ tons. (Fill in the blank.)

3 tons A good guideline when selecting a cooling system for an older home is to assume that approximately 1 ton of cooling capacity will be necessary for each 500 ft2 of living space. This old farmhouse presumably has drafty windows and lots of air infiltration, so a 3-ton unit would be a reasonable choice If this project were a residence of similar size but new construction, a more appropriate guideline would be about 1 ton for each 1,000 ft2 of floor area. New construction materials and methods, such as house wrap and vapor barriers, additional insulation, improved windows, caulking and sealants, and so on, make it much easier to control the environment within the residence and require less cooling capacity

The water pressure at a water main connecting to a building is 47 lbf/in². At a 32 foot elevation above this point what is the pressure? Ignore friction.

33 psi Pressure Loss = = (32') (.433 lbf/in²) = 13.856 Pressure at 32' = = 47 lbf/in² - 13.856 lbf/in² = 33.144 lbf/in²

What is the minimum required clear width between handrails of an accessible ramp?

36" According to section 405.5 of the 2010 ADA standards, the minimum required clear width between the handrails of an accessible ramp is 36″

At the base of a house the ground elevation is +365.12. What is the ground elevation if the grading is sloped at 1% away from the base of the house for 6'-0"?

365.06 Use proportions to solve the problem x/6′-0″ = 1/100 x = (1)(6'-0″)/100 x = 0.06 Final height = = 365.12 - 0.06 = 365.06

Any opening in a guard shall be less than what dimension?

4 inches According to §1013.4 of the 2012 IBC: "Required guards shall not have openings which allow passage of a sphere 4 inches (102 mm) in diameter from the walking surface to the required guard height" Note that the code states a 4 inch sphere shall not be allowed to pass through the guard. This means that any openings in a guard must be smaller than 4 inches

The minimum distance between handrails on a stair that is part of an accessible means of egress is ________

48 inches Section 1007.3 Stairways of the IBC states: "In order to be considered part of an accessible means of egress, a stairway between stories shall have a clear width of 48 inches minimum between handrails..."

Bids come back for a project. The project is a 25'-6" x 60'-0" ballroom addition. The project comes in $53,550 over budget. The client loves the design and wants to keep the finishes, but shrink the ballroom in the long direction to adjust to the budget. If the cost per square foot is $175 what is the new length of the building?

48' To find the amount to lose we calculate $53,550 = (25.5')(x)($175) x = 12 ft Hence, 60'-0" - 12'-0" = 48'

An 8'-0" solid column with a 6" diameter stretches 0.125" due to a 25 kip tensile load. What is the modulus of elasticity of the column?

679,061 psi E = = stress / strain = (Force/Area) / (Change-in-Length/Length) = (25,000 lbs/pi3in^2) / (.125" / 96") = 679,061 psi

A balcony is partially supported from structure above by a steel rod with a 1.25" diameter. The load on the rod is 10,000 lbf. What is the stress in the rod?

8,130 psi The radius of the rod is .625". The area of the rod is 1.23 in2 Stress = f = = P/A = (10,000 lbf) / (1.23 in2) = 8,130 psi Other types of stresses consist of torsion, bending, and combined stresses. Torsion is a type of shear in which a member is twisted. Bending is a combination of tension and compression like the type that occurs in beams. Combined loads can occur in many situations. For example, a column resisting loads from above and lateral wind loads is subjected to both compression and bending

If a load of 12,000 lbf is applied to a 3 ft long, 4x4 Douglas fir no. 2 wood column, how much will it compress? The modulus of elasticity for Douglas fir no. 2 is 1,700,000 psi.

= 0.021" Strain = e = = PL/AE = ((12,000 lbf) ( 3ft)) / ((3.5")(3.5")(1,700,000 psi)) = 0.021"

If the target illuminance in a 10'-0"x13'-3" room is 25 fc what should the lumen output be in that room? Assume a CU of 0.6 and an LLF of 0.7.

For this question we use the formula: Footcandles (fc) = = (lumens*CU*LLF) / (room square feet) lumens = = (fc*area) / (CU*LLF) = (25 * 10'-0" * 13'-3") / (0.6 * 0.7) = 7887 lumens

What is used to minimize corner chipping of concrete? (A) chamfer strips (B) hardeners (C) rustication strips (D) walers

(A) chamfer strips A chamfer strip is a small, triangular piece of material placed in the corners for forms to prevent sharp 90 degree corners, which are difficult to cast and have a tendency to break off during use or when the forms are removed

Which MasterFormat division would include the specification requirements for metal studs for interior partition walls? (A) 05 (B) 09 (C) 10 (D) 13

(B) 09 Light-gauge metal framing for interior partitions is specified in Division 09, Finishes. Metal studs are typically specified as a part of gypsum board assemblies

According to the American Concrete Institute's Building Code Requirements for Structural Concrete (ACI 318), a cantilevered reinforced concrete slab spanning 10 ft needs to have a minimum thickness of (A) 6" (B) 12" (C) 20" (D) 24"

(B) 12" According to ACI 318 table 9.5(a), a cantilevered reinforced concrete slab of span L must have a minimum thickness of L/10 h = = L/10 = (10ft)(12in/ft)/10 = 12"

What type of brick would most likely be specified for an eastern exposure in New Hampshire (A) NW (B) FBX (C) MW (D) SW

(D) SW SW stands for severe weathering and would be the type that should be specified for the Northeast NW is negligible weathering MW is moderate weathering FBX refers to the finish appearance

Which of the following is the best sealant to use between exterior precast concrete wall panels? (A) acrylic (B) butyl (C) latex (D) polyurethane

(D) polyurethane Polyurethane sealant, either one-part or two-part, provides excellent resistance to weather and is capable of 25% to 50% movement. It can span the wide joints typical of precast concrete, is available in colors, and can be painted Acrylics are unsuitable for this situation because of their limited potential for joint movement and their inability to fill the large-width joints that are typical of precast concrete Butyls are unsuitable because of their limited joint movement and because they are only available in darker colors. They are generally used for areas underwater Latex sealants also have limited joint movement capability and are typically used for joints with no expected movement, such as those around door and window frames

A main trunk duct is to be placed in the interstitial space above a suspended ceiling and below the structural framing. The space is not constricted. When capacities are equal, which of the following shapes of ducts would be best to use? (A) rectangular, with the long dimension horizontal (B) rectangular, with the long dimension vertical (C) square (D) round

(D) round A round duct is the most effective choice and offers the smallest possible perimeter for the same cross-sectional area, thus minimizing friction and pressure loss. A square shape would use the available space most efficiently, but a duct of this shape is not as efficient overall as a round duct As ducts become more rectangular, they become less efficient and have increased friction loss. A rectangular duct with the long dimension horizontal would only be used if space was a problem

In wind design, what is the permissible drift of one story related to an adjacent story in a building where the story height is 12 ft? (A) 0.036" (B) 0.36" (C) 0.72" (D) 1.1"

in wind design, the maximum permissible drift of one story relative to an adjacent story is 0.0025 times the story height. The story height is 12 ft, and the maximum permissible drift is d = = 0.0025h = 0.0025 (12in)(12in/ft) = 0.36"

An architect is surveying an existing building to determine if the facility complies with International Building Code (IBC) requirements for accessibility. Which of the following elements must be modified to satisfy the code requirements? (A) fire extinguisher cabinet with a "bubble" door that protrudes 5" from the wall (B) 1/4" beveled threshold at the main entrance doors (C) clear floor area 24" wide on the pull side of the door to the men's restroom from the hallway, which is approached from the front (D) ramp from the original building to a later addition that rises 6" over 8'

(A) fire extinguisher cabinet with a "bubble" door that protrudes 5" from the wall A wall-mounted object placed no more than 27" above the floor cannot protrude more than 4" from the wall

Water enters a supply system from a water main with a pressure of 45 lbf/in2. Ignoring loss due to friction, the pressure available to operate a fixture located 40 ft higher than the main is (A) 28 psi (B) 31 psi (C) 35 psi (D) 38 psi

(A) 28 psi The pressure loss between the main and the fixture is equivalent to 40 ft of static head. Use a conversion factor of 0.433 psi per foot of static head Ploss = (40ft)(0.433 lbf/in2/ft) = 17.32 lbf/in2 Ignoring friction loss, the water pressure at the fixture is 45 - 17.32 = 27.68 lbf/in2 = (28 psi)

An existing partition separating two rooms is determined to be insufficient for reducing sound transmission. The partition consists of 4" metal studs spaced 24" on center with a single layer of 5/8" gypsum board on each side. There are no penetrations in the partition. To improve the transmission loss of the partition in the most economical way, which of the following modifications should the architect recommend? (A) Add resilient channels to one side of the wall and attach a single layer of gypsum board to the other side (B) Add sound-absorbing panels to the noisy side of the partition, and add an additional layer of gypsum board to the opposite side (C) Remove one layer of gypsum board, install sound-attenuating insulation, and replace the wall finish with a sound deadening board and a finish layer of gypsum board (D) Cover one side of the partition with an additional layer of gypsum board, and add two additional layers of gypsum board to the other side

(A) Add resilient channels to one side of the wall and attach a single layer of gypsum board to the other side The best way to improve the transmission loss is to add mass and resiliency to the partition. This can be accomplished economically by adding extra gypsum board and mounting one layer on resilient channels Sound-absorbing panels would not affect the transmission loss; they would only affect the noise reduction in the room on the other sire where the panels were installed. Removing the wall finish would not be the most economical method for the results obtained by adding insulation and then replacing new wallboard over sound-deadening board. Adding the extra mass of three layers of gypsum board would not be as effective as using resilient channels with two additional layers of gypsum board as in option (A)

Which statement concerning fire-rated door assemblies is correct? (A) Hinges must always be the ball-bearing type (B) Under some circumstances a closer is not needed (C) Labeling is required for the door only (D) The maximum width is 3'-0"

(A) Hinges must always be the ball-bearing type A closer is always required Labeling is required for the door and frame The maximum width of a door leaf is 48", however there are various exceptions to this in the IBC chapter 10

What type of mortar should be specified for a concrete masonry unit (CMU) foundation wall? (A) M (B) N (C) O (D) S

(A) M Type M mortar is high-strength mortar. It is suitable when masonry is subject to high lateral loads, compressive loads, severe frost, or masonry below grade. It should be specified for exterior applications at or below grade Type N or S is the best for exterior applications above grade and for interior load-bearing walls Type O is well suited for interior and protected exterior non-load-bearing partitions

According to the IBC, a soft story is a story in which the (A) lateral stiffness is less than 70% of that in the story above or less than 80% of the average stiffness of the stories above (B) lateral stiffness is less than 60% of that in the story above or less than 70% of the average stiffness of the stories above (C) lateral stiffness is less than 50% of that in the story above or less than 60% of the average stiffness of the stories above (D) story lateral strength is less than 80% of that of the story above

(A) lateral stiffness is less than 70% of that in the story above or less than 80% of the average stiffness of the stories above Option (B) describes an extremely soft story Option (D) describes a weak story

How are the diagonal members of the X-bracing of a tall structure normally designed to minimize costs? (A) One diagonal brace is designed to be stressed in tension while the other is not stressed (B) One diagonal brace is designed to be stressed in compression while the other is not stressed (C) Both diagonal braces are designed to be stressed in tension at the same time (D) Both diagonal braces are designed to be stressed in compression at the same time

(A) One diagonal brace is designed to be stressed in tension while the other is not stressed One diagonal brace is normally designed to be stressed in tension while the other is not stressed. This is done to minimize costs. Diagonal braces are not designed to work in compression. By designing diagonal members, their size and therefor their cost is minimized. X-bracing is a common type of lateral bracing for tall structures. Bracing is often placed at the center of the structural framing, such as around the building's central core. When a wind load hits a building from one side, one of the braces acts in tension and the other one is not stressed. When the wind direction is reversed, the brace that works in tension is reversed accordingly while the other is not stressed

Which of the following statements about tempered glass is true? (A) Tempered glass is required in entry doors (B) Tempered glass is not a requirement for sidelights with sills below 18" (C) Tempered glass is not a retirement for glazing within 1' of doors (D) Tempered glass is required in glass within 36" of doors

(A) Tempered glass is required in entry doors Safety glazing is required in all areas subject to human impact. This includes glass doors and any glass within 24" of doors. Glass farther than 24" from doors and with a sill over 18" above the floor does not have to be safety glazed

Which type of masonry cement mortar has the highest compressive strengths? (A) Type M (B) Type N (C) Type O (D) Type S

(A) Type M Type M mortar has a minimum average 28-day compressive strength of 2500 psi Type O has the lowest compressive strength, 350 psi. It is only used for light loads where freezing is not expected For exterior walls and interior walls under normal loads, Type N mortar is commonly used Type S mortar is used for heavier loading on interior walls and for exterior walls at or below grade, such as foundation walls, retaining walls, pavements, walks, and patios When high-strength mortar is required for heavy loads or for cases where the mortar will be exposed to severe, saturated freezing, Type S or Type M mortar is used

Which of the following statements are true about electronic ballasts installed on fluorescent lamps? (Choose the two that apply.) (A) Flicker less than with conventional ballasts (B) Lamps cannot start in temperatures below 40 degrees F (C) Noise is greater than with conventional ballasts (D) Lamps cannot be dimmed (E) Existing fixtures with conventional ballasts can be retrofitted with electronic ballasts (F) Electronic ballasts are less energy efficient

(A) and (E) Electronic ballasts have many advantages over conventional ballasts. Many annoyances associated with fluorescent lamps, such as humming and flickering, are greatly reduced or eliminated with electronic ballasts. Electronic ballasts permit lamps to be operated at a wider range of temperatures, down to about 0F, and let lamps be dimmed more easily and economically. In addition, the ballast itself is smaller and lighter in weight and more energy efficient. Existing fixtures with conventional ballasts can be retrofitted with electronic ballasts to realize the advantages of newer technology

An architect consults with an engineering firm that provides mechanical, electrical, and plumbing services, as well as lighting design. Who is responsible for verifying that recessed downlights do not interfere with the ductwork shown on the plans? (A) architect (B) electrical engineer (C) lighting designer (D) mechanical engineer

(A) architect The architect is responsible for the overall coordination of all the contract documents prepared by his or her consultants If the owner chooses to hire engineers outside of the architect's contract, or elects to have some of this work performed by a contractor on a design-build basis, the owner then becomes responsible for coordination of the documents

A two-way concrete slab is normally reinforced for (A) bending moment in both directions (B) bending moment in the short direction only (C) bending moment in the long direction only (D) temperature/shrinkage stresses in both directions only

(A) bending moment in both directions A two way concrete system is somewhat square in shape and reinforced for bending moment in both the short and long directions In the one-way slab, the reinforcement is placed for bending moment in the short direction, while the long direction is reinforced at a minimum ratio for temperature/shrinkage stresses

Which of the following devices would best control entry to a secure laboratory? (A) card reader (B) central station alarm (C) photoelectric cell (D) ultrasonic detector

(A) card reader A card reader is one type of security device that is used to control access. Security systems are generally comprised of access controls, notification devices, and intrusion detectors A central station alarm is a method of notification Photoelectric cells and ultrasonic detectors are devices used for intrusion detection

The moment-resisting frames in the IBC generally can be used in which of the following seismic design categories? (A) categories A, B, and D (B) category D only (C) category E (D) category F

(A) categories A, B, and D Intermediate moment-resisting frames are generally not permitted in seismic design categories D, E, or F. However, steel intermediate moment-resisting frames with heights up to 35 ft can be used in category D. The design requirements of the intermediate moment-resisting frames are less stringent compared to the special moment-resisting frames. The latter type must be ductile and must satisfy certain provisions of the IBC

A positive bending moment at a section of a beam implies (A) compression stress in the top fibers of the beam and tension stress in the bottom fibers (B) tension stress in the top fibers of the beam and compression stress in the bottom fibers (C) compression stress in all fibers of the section (D) that the bending moment is not related to tension and compression stresses

(A) compression stress in the top fibers of the beam and tension stress in the bottom fibers Sign conventions identify both the bending moment that creates compression stress in the top fibers of a beam and tension stresses in the bottom fibers of a beam as a positive bending moment

On floors subject to deflection granite installations should include a (A) membrane (B) latex additive in the mortar (C) type of thinset mortar (D) sand cushion

(A) membrane A membrane is part of a total assembly that also includes reinforcing and a thick bed of mortar on which the granite is laid. The membrane allows the structural slab to move independently of the finish flooring so that any deflection does not crack the floor

Which type of test is used to determine the required size of a leaching field? (A) percolation (B) yield (C) aquifer (D) water table

(A) percolation The required size of a leaching field is determined by the quantity of effluent that must be accommodated and the ability of the soil to let the effluent soak in. This permeability of the soil is measured by the percolation test. None of the other options test either quantity or permeability

Adding a shear key to the base of a cantilever retaining wall helps to prevent (A) sliding of the wall (B) overturning of the wall (C) breaking and cracking of the wall base (D) simultaneous breaking and overturning of the wall

(A) sliding of the wall Adding a shear key to the base of a cantilever retaining wall helps prevent sliding of the wall by increasing the contact surface, and therefore the friction forces, between the base of the wall and the soil

Which of the following is required for an access floor system? (Choose the three that apply.) (A) lifting devices (B) modular panels (C) pedestals (D) stringers (E) acoustical batts (F) handrails

(A), (B), (C) Access floor systems may use stringers, which are rigid connections between pedestals, but they can also be the stringer-less type, which rely only on pedestals and panels to keep systems in place All access floors use modular panels set on pedestals of some type. Removal of the panels requires a lifting device

Which of the following factors affect the allowable shear value in the diaphragm system made of steel deck? (Choose the four that apply.) (A) thickness of the steel deck (B) size of welds and other connections between deck and framing (C) spacing of welds and other connections between deck and framing (D) presence of concrete topping (E) the location of steel stud partitions above and below the steel deck (F) whether it is used in a steel frame or concrete frame building structure

(A), (B), (C), (D) The location of steel stud partitions above and below the steel deck, as well as whether the system is used in a steel frame or a concrete frame building structure do not affect the allowable shear value The thickness of the steel deck, the size of welds and other connections between the deck and framing, the spacing of welds and other connections between deck and framing, and the presence of concrete topping have an effect on the allowable shear value of a diaphragm made of steel deck

An architect is planning a 100,000 ft2 university classroom building. The mechanical engineer estimates that the total floor area required for the boiler room and the chilled water plan will be about 3,000 ft2. Which of the following criteria should also be kept in mind when determining the location and design of the mechanical rooms? (Choose the three that apply.) (A) Each mechanical room should have at least one exterior wall (B) The boiler room should be adjacent to the chilled water plant (C) Rooms should be as square as possible (D) Ceilings in both rooms should be at least 12' high (E) Mechanical rooms must be placed on the ground floor (F) The mechanical rooms should be equal in size

(A), (B), (D) Boiler rooms and chilled water plants should be located adjacent to one another when possible; in some buildings, the two functions are placed in the same room. It is imperative that the rooms each have at least one exterior wall to permit access to fuel tanks that may be located outside and to allow for adequate ventilation Recommended ceiling heights vary depending on the type of equipment chosen, but generally 12' is the minimum The rooms should be long and narrow rather than square and sized to best accommodate the equipment Boilers and chillers are heavy and require additional structural support. It is often most economical to locate them on the ground floor, but this is not required. They tend to be noisy, so the mechanical rooms should be placed in locations within the building where the noise will not disrupt critical tasks. Soundproofing techniques should also be integrated to acoustically separate the mechanical rooms from the occupied spaces

Which of the following is a requirement of the National Fire Protection Association's Standard for Portable Fire Extinguishers (NFPA 10)? (Choose the three that apply.) (A) Fire extinguisher cabinets must have a vision panel or be clearly marked with a sign (B) Fire extinguishers must be tested regularly and have an approved label (C) Fire extinguisher cabinets may not protrude into the hallway more than 4" (D) When fire extinguishers are required, no occupant may be more than 75' from a fire extinguisher (E) Fire extinguishers must be mounted at 48" above the finish floor (F) Cabinets must be locked at all times

(A), (B), (D) Extinguishers must be clearly marked and visible; located no more than 75' from each building occupant; properly maintained, tested, and labeled; and readily accessible in case of an emergency A fire extinguisher cabinet must not protrude more than 4" into the hallway and mount no higher than 48", but this is a requirement of the ADA, not NFPA

Which of the following statements about precast concrete are true? (Choose the three that apply.) (A) Type III cement (high-early-strength) is often used for precast concrete members (B) Precast concrete members can be a maximum of 14' wide (C) Concrete used in precast members is typically 3000 psi (D) Conditions are more controlled during the production of precast concrete members than they would be for similar structural systems built in the field (E) Precast concrete can be used only with white aggregate (F) Precast concrete shapes include flooring planks, wall panels, and door modules

(A), (B), (D) Wrong: Concrete used in precast members is generally a higher-strength mix than the 3000 psi concrete typically used for site cast applications Precast concrete can have many different appearances by using colored or gray aggregate, formed surfaces or patterns. Precast concrete shapes include floor planks, beams, columns, wall panels, and domes Correct: Type III (high-early-strength) cement and steam curing allows pre-stress plants to get finished beams and tees out of the beds and into the yard more quickly so that production can continue. They are often then warehoused in the yard until they have passed the 28-day cylinder testing Precast members are generally transported over the highway, so their width is limited to the width of a travel lane In general, conditions can be better controlled during the production of precast concrete than they can in the field. Forms can be used repeatedly, and curing can take place under shelter and in controlled conditions, making precast concrete products both economical and consistent

What is used to keep water from penetrating an expansion joint at the intersection of a roof and a wall? (A) base flashing (B) counter flashing (C) sealant (D) coping

(B) counter flashing Base flashing extends from the roof over the cant strip and up the wall. Counter flashing covers the base flashing to extend from the wall over the base flashing and to cover any expansion joint that may occur at this point. Coping protects the top of the parapet. Sealants by themselves are not adequate to cover a major expansion joint as would occur at the roof and wall intersection

The application of epoxy-coated reinforcing bars would be specified in which type of location? (Choose the four that apply.) (A) parking garage (B) fishing pier on the ocean (C) interior column in an office building (D) exterior concrete staircase (E) cold climate highway paving floor slab (F) floor slab

(A), (B), (D), (E) Epoxy-coated reinforcing bars are used when the concrete will be exposed to chlorides (salts) such as deicing salts and those in seawater. The epoxy coating helps keep the salts from corroding the steel by chemical reaction. An interior column or floor slab is unlikely to be in contact with chlorides, so it is not necessary to specify epoxy-coated rebar in this application

When designing a beam, why is it necessary to control deflection? (Choose the four that apply.) (A) Excessive deflection is visually disturbing (B) Excessive deflection can break adjacent windows (C) Excessive deflection is a sign that the beam is unsafe in bending (D) Excessive deflection is a sign that the beam is unsafe in shear (E) Excessive deflection can crack adjacent partitions (F) Excessive deflection can crack adjacent plaster

(A), (B), (E), (F) Excessive beam deflection is visually disturbing and might give the impression that the beam is not safe when it is. An excessive deflection could also damage adjacent building materials, breaking windows, or cracking partitions and suspended plaster ceilings A beam can have a large deflection and still be sage in bending and shear

The IBC requires that a certain amount of accidental torsion be considered even when a building is symmetrical. Why? (Choose the four that apply.) (A) to allow for nonuniform vertical loading (B) to allow for asymmetrical floor openings (C) to reduce the effect of overturning moments (D) to reduce drift (E) to allow for eccentricity in rigidity due to nonstructural elements and seismic ground motion (F) to allow for the fact that positions of loads in an occupied building cannot be exactly determined

(A), (B), (E), (F) The accidental torsion is not related to either the overturning moment or drift. The code requires that the mass at each level of the building be assumed to be displaced in each direction a distance equal to 5% of the building dimension at that level in the direction perpendicular to the direction of the force

Which of the following would be appropriate for fire protection in an elementary school? (Choose the three that apply.) (A) ionization detector (B) photoelectric detector (C) temperature rise detector (D) gas-sensing detector (E) flame detector (F) laser beam detector

(A), (B), (F) A temperature rise detector would not give early warning to the occupants. A gas-sensing detector by itself would not be appropriate for this application. A flame detector responds to infrared radiation given off by flames and may respond too late to give adequate warning If properly located, either an ionization or photoelectric detector would work. Ionization detectors sound an alarm when they sense the products of combustion. Photoelectric detectors monitor smoke. Either would sound an alarm before a temperature rise detector. A laser beam detector is a type of photoelectric detector and would also be appropriate

Software utilizing building information modeling (BIM) provides a digital, spatial, and measurable design tool that may be utilized by the entire design and construction team. This is intended to provide which benefits over a non-BIM design method? Choose the three that apply.) (A) collaboration with design team earlier in the design development phase (B) license sharing with consultants so additional software need not be aquired (C) utilization of intelligence within models to generate schedules and cost estimates (D) use of models to facilitate ongoing building maintenance and operations (E) reduction of time needed to design buildings (F) elimination of consultants because of the software's ability to analyze BIM data

(A), (C), (D) Early development of the model is shared with the design team with the intent of identifying issues and locating potential clashes between disciplines. The model intelligence can provide data that is easily incorporated by the software into schedules and cost estimates. The BIM model also may contain data such as installation time, cost, manufacturers' details, sustainability, and maintenance information. Utilizing this information will provide an opportunity for building operations to use the BIM model in maintaining and operating the building long term

Which of the following will effectively reduce the possibility of termite infestation? (Choose the three that apply.) (A) Design the slope of the grade near the foundation to fall away from the structure (B) Specify pressure-treated lumber (C) Require that soil poison be applied to the footprint area of the building before construction operations begin (D) Provide a gravel drainage area where the foundation wall meets the surrounding grade (E) Wood members are located above the ground (F) Provide adequate attic ventilation

(A), (C), (E) There is no way to completely prevent termites from entering a structure, but there are many elements that architects can include in their designs to make the environment less hospitable to instects Termites, along with carpenter ants, bees, and powderpost beetles, flourish in moisture and wood, and they generally enter buildings at the ground level. Preventing wooden parts of the structure from coming in contact with the ground is one of the keys. Foundations should also be kept as dry as possible; designing the grade to fall away from the building will help to keep the area well drained. It is important to specify that an appropriate soil poison be applied to the area of the building footprint before construction begins. The type of insecticide used varies depending on the type of insect it is to combat Pressure-treating lumber makes the wood more resistant to damage from water but does not make it more resistant to insect damage One or the most common errors building owners make is to pile mulch around the base of shrubbery planted at the perimeter of the building. The much, whether made of pine bark, cocoa bean shells, gravel, or any other material, holds in the moisture, which is good for plants but bad for the structure. Termites thrive in the warm, moist soil underneath the gravel and can use that as an access point to enter the basement or crawlspace. Using a gravel drainage area in lieu of gutters and downspouts can have the same consequences if the system is not designed to carry the water away from the perimeter of the building

Two elevators are on the north side of a lobby and two elevators are on the south side of a lobby. What should the minimum distance between the two sets of elevators be? (A) 8 ft (B) 10 ft (C) 12 ft (D) 14 ft

(B) 10 ft The minimum width of the lobby should be 10 ft. A 10 ft wide lobby will allow sufficient space for a group of passengers to gather, but is small enough that person in the lobby can see all of the elevators while waiting for an available car

In calculating solar heat gain, what value must be known in addition to the area of the glass? (A) mean radiant temperature (B) design cooling factor (C) equivalent temperature difference (D) coefficient of heat transfer

(B) design cooling factor The design cooling factor and the area of the glass must both be known to calculate solar heat gain Equivalent temperature difference is used to calculate heat gain through the building envelope, such as walls and roofs

A spotlight shining perpendicularly to a wall 15' away has a candlepower output of 3500 cd. The wall finish is paint with a reflectance of 75%. What is the luminance of the wall where the wall is perpendicular to the direction of light? (A) 4.90 fL (B) 11.7 fL (C) 15.6 fc (D) 55.7 fc

(B) 11.7 fL The footcandle (lux) level of light shining on the wall must be determined first. Because the direction of the light is perpendicular to the wall, the inverse square law is used E = = I/d^2 = 3500 cd / 15ft^2 = 15.56 fc Once the footcandle (lux) level is determined, it is multiplied by the reflectance to find the brightness. Reflectance is the ratio of reflected light to incident light. The former is measured in footlamberts (15.56 fc) (0.75) = 11.7 fL

Which of these statements about reinforcing bar (rebar) sizes is true? (A) American and metric sizes are now based on a unit-less number (B) American bar numbers equal the number of eights of an inch across the diameter (C) Reinforcing bars are available only in Grade 40 (D) Epoxy-coated reinforcing steel bars are used only on interior concrete floor slabs

(B) American bar numbers equal the number of eights of an inch across the diameter The designation system for American standard sizes of rebar is based on the number of eighths of an inch in the nominal diameter of a bar, up to 1". So a #3 bar, is 3/8 in diamter Epoxy-coated rebar is typically used in exterior concrete construction to slow down or prevent the steel from rusting Steel reinforcing bars are typically manufactured in grade 40, grade 60, and grade 75

Which strategy will most effectively reduce stormwater runoff from a site (A) Capture rainwater for irrigation and other nonpotable uses (B) Reduce the amount of impervious area (C) Construct a garden roof (D) Construct an on-site stormwater treatment facility

(B) Reduce the amount of impervious area Reducing impervious area on a site is the most effective way to reduce stormwater runoff. Impervious area is any area of building or paving that does not allow stormwater to seep into the ground Though less effective, installing a garden roof is also a way to reduce stormwater runoff Reusing captured rainwater for other approved uses, such as flushing toilets and irrigating landscaping, is a good way to manage stormwater and keep it on site An on-site treatment facility would help remove contaminants from stormwater before it is released into nearby rivers or streams, but would not reduce stormwater runoff

The pyramidal forms of Manhattan skyscrapers built in the early to mid-1900s were the result of (A) the influence of European tastes and style on American architecture (B) a response to zoning laws governing setbacks and height restrictions (C) an attempt to make buildings safer in case of fire by housing fewer people on the uppermost floors (D) the structural limitations of iron as a building material

(B) a response to zoning laws governing setbacks and height restrictions In 1916, New York City passed a zoning law designed to protect access to light and ventilation for all buildings. Skyscrapers were being built as quickly as possible, and New Yorkers feared that these walls of masonry and steel reaching into the sky would make the street level cold, dark, and cavernous. As buildings rose higher and higher, they were required to comply with more stringent setbacks. In order to build the maximum rentable area on their extremely valuable sites, developers adopted the stepped, wedding cake-like shape

A slip joint is used in the head of an aluminum storefront system to (A) allow for expansion and contraction (B) accommodate the deflection of the structure (C) facilitate the installation of the mullions (D) provide a way to install and remove the glazing

(B) accommodate the deflection of the structure Any deflection of the structure above a storefront could possibly break the glass or bend the mullions. A slip joint is used to prevent the weight of the structure above from bearing on the framing or glazing

Codes limit the number of conductors permitted in a conduit for which of the following reasons? (Choose the two that apply.) (A) to maintain maximum ampacity (B) to control heat buildup in the conduit (C) to minimize problems with harmonic currents (D) to prevent damage to the conductors when they are pulled through the conduit (E) to keep conductors on the same circuit (F) to allow room in the conduit for data wiring

(B) and (D) Too many conductors carrying too much current in an enclosed area can generate excessive heat. In addition, conductors can be damaged if too many are pull through a small conduit. For these reasons, the National Electrical Code (NEC) limits the number of conductors permitted in a conduit The NEC only requires that the ampacity of the conductors be derated if the number of conductors in a raceway or conduit exceeds three. Harmonic currents are only a problem with unconventional electrical loads such as computers, electronic lighting ballasts, and other electronic equipment. When these types of loads are supplied by conductors, the neutral conductor must be counted as one of the three allowable conductors in a conduit before ampacity is derated

Where would a sound intensity level of 120 dB be found? (A) in a sewing factory (B) at a rock concert (C) in an architect's open plan office (D) during nap-time at a nursery school

(B) at a rock concert 120 dB is almost deafening and be felt throughout a listener's body. It can cause ringing in the ears and temporary loss of hearing Common sound intensity levels range from 0 dB, the threshold of hearing, through 130 dB, the threshold of pain

To achieve a slip resistant finish on a concrete floor slab, the architect should specify a (A) float finish (B) broom finish (C) hard steel-troweled finish (D) light steel-troweled finish

(B) broom finish For slip resistance, a broom finish is the best choice A float finish is a rough finish intended for outdoor surfaces and interior slabs that will become the substrate for a finish material that does not require a perfectly smooth underlayment, such as carpet or tile The troweled finishes are very smooth and can be sealed or painted to become the final floor finish, or they can be the substrate for a finish material that requires a perfectly smooth surface, such as vinyl composition tile

Blocking that eases the transition between a roof deck and the parapet wall in a membrane roof installation is known as a (A) chamfer (B) cant (C) transition strip (D) scupper

(B) cant A cant strip is an angled piece of blocking that eases the transition between the roof deck and the parapet wall when installing a roofing system so that the membranes or roofing felts do not crack or split when they are applied. In addition, the slope helps the water to drain away from the joint and allows the flashing and membranes to be lapped in a way that keeps water from entering

The energy required to turn a raw material into a finished building product is known as (A) manufacturing energy (B) embodied energy (C) production energy (D) commissioning energy

(B) embodied energy The amount of embodied energy in a building material is a measure of the resources that went into its acquisition (mining, harvesting, etc), processing (transportation, the mechanical process necessary to convert the material from one form to another, etc), and final production and delivery (packaging, transportation, etc)

Asphalt-impregnated building paper is used under siding primarily to (A) improve thermal resistance (B) increase the water resistance (C) act as a vapor barrier (D) all of the above

(B) increase the water resistance Although asphalt-impregnated paper can act as a vapor barrier, the fact that it is placed on the outside of the sheathing precludes options (C) and option (C) from being correct. It does add a little to the thermal resistance, but its primary purpose is to prevent any water that seeps behind the siding from getting into the structure. It also serves to prevent air infiltration

Materials or assemblies with a high noise reduction coefficient (NRC) generally have a (A) high sound transmission class (STC) (B) low sound transmission class (STC) (C) high reverberation time (D) low absorption coefficient

(B) low sound transmission class (STC) Materials or assemblies with a high NRC generally have a low STC. The NRC, or noise reduction coefficient, is a measure of how absorptive a material is to sound. Materials with a high NRC are generally very porous materials such as acoustical ceiling tile, fabrics, carpet, and so on. The STC, or sound transmission coefficient, measures how well a material blocks sound transmission from one space to the next The absorption coefficient is linked to the NRC, but they are directly proportional; high NRCs equal high absorption coefficients. Reverberation time is a calculation for a space rather than a property of a material or an assembly

Which type of high intensity discharge (HID) lamp must be installed in a specified burning position? (A) mercury vapor (B) metal halide (C) high-pressure sodium (D) low-pressure sodium

(B) metal halide Metal halide lamps are particularly sensitive to orientation, and they lose efficiency and lumen power if not installed correctly. All metal halide lamps are designed with a proper burning position: base-up, base-down, horizontal, or universal The other types of HID lamps do not have this characteristic and can be installed in any orientation

An architect is designing an art school at a major university. Which combination of daylighting and electric lighting would be the most appropriate choice for the painting studios? (A) south-facing windows and incandescent recessed lights (B) north-facing windows and skylights and fixtures with high color rendering index (CRI) fluorescent lamps (C) windows to the east and west and fluorescent fixtures in coves at the perimeter of the studio (D) north-facing skylights and metal halide lamps at each workstation

(B) north-facing windows and skylights and fixtures with high color rendering index (CRI) fluorescent lamps Art studios require optimal color rendering and even daylight. The best combination of natural and artificial lighting techniques would be north-facing windows and skylights along with the best quality high-color rendering lamps that the budget will allow. Task lighting should also be provided for the students' work areas

The standardized levels of finish in the gypsum wallboard industry refer, among other things, to the (A) quality of workmanship of the final finish (B) number of coats of joint compound used (C) thickness of joint compound used (D) type of texturing used

(B) number of coats of joint compound used Their are six levels of finish according to the Gypsum Association. One requirement for these levels is the number of coats of joint compound used. The levels are 0, 1, 2, 3, 4, and 5. Level 0 requires no taping, finish, or accessories, while Level 5 requires three coats of joint compound over joints and fastener heads, as well as a final skim coat over the entire surface of the wall

An architect is designing an addition to a high school to house a new gym and locker rooms. The architect plans to construct the exterior walls with concrete masonry units and apply an exterior insulation and finish system (EIFS) over the block. The new gym will be located adjacent to the school's baseball field. To take advantage of the material's insulation properties and provide the best impact resistance to avoid dents from fly balls, which type of EIFS should be specified? (A) polymer based (PB) (B) poly-modified (PM) (C) mineral based (MB) (D) expanded polystyrene (EPS)

(B) poly-modified (PM) Polymer-modified (PM) mineral-based systems have high impact resistance and provide good insulation. They consist of a base and finish coat of synthetic stucco applied over extruded polystyrene (XPS) insulation board Polymer based (PB) systems are made up of a very thin base coat of portland cement and polymer over fiberglass mesh with a thin finish coat of polymer-based synthetic stucco over expanded polystyrene (EPS) insulation board. They are lighter in weight than PM systems, but because their plaster coats are so thin, they do not resist impact well Mineral based (MB) systems are basically conventional three-coat portland cement stucco systems. They are very impact resistant, but since the stucco is not applied over an insulation board, the system does not offer the insulation of PB and PM systems

An architect is writing specifications for a small clothing boutique. The architect has worked closely with the owner and fashion designer to plan an intricately detailed tile floor for the main showroom. The designer wishes to use a type of marble she saw installed in a friend's home in Rome. Which type of tile specification is most appropriate (A) prescriptive (B) proprietary (C) descripteive (D) reference standard

(B) proprietary A proprietary specification is appropriate in this situation because it gives the architect the most control over the product provided and installed by the contractor. The architect and owner will have the opportunity to select the products they want to use for the floor and will refer to those specific products in the specification A proprietary specification is a type of closed, or prescriptive, specification Descriptive and reference standard specifications are types of open specifications. They outline the final results desired but do not specifically tell the contractor what materials to use Descriptive specifications require the architect to list all of the desired characteristics of the material and put the onus on the contractor to find a product that will satisfy the requirements. They can be difficult to write because the level of detail that must be included to ensure that an appropriate product is chosen A reference standard specification is much simpler to write. It refers to industry standards to define the desired characteristics of materials and installation systems

If not connected to other sections, the section of the soil stack that is located above the highest plumbing fixture in a building is called the (A) vent stack (B) stack vent (C) cleanout (D) vacuum breaker

(B) stack vent The stack vent is the portion of the soil stack above the highest fixture. It serves as a vent for the stack and is open to the outside at the top A vent stack is a collection of vents from a number of fixtures that share one exterior outlet A clean-out is an area of the plumbing that can be accessed to clear obstructions from the system A vacuum breaker is a flap that opens to admit air if there is suction in a water pipe, which prevents siphoning of wastewater back into the clean water supply

What roof live load should be used to design a column that supports 450 ft2 of a roof with a slope of 5" of rise per foot of run? Calculate the reduction factors for tributary area and slope, and then find the design roof live load. (A) 6.35 psf (B) 9.53 psf (C) 14.3 psf (D) 21.5 psf

(C) 14.3 psf R1 = = 1.2 - 0.001 (Area) = 1.2 - 0.001 (450ft2) = 0.75 R2 = = 1.2 - 0.05 (Roof rise / 12") = 1.2 - 0.05 (5") = 0.95 Lr = = (20 lbf/ft2) (R1) (R2) = (20 lbf/ft2) (0.75) ( 0.95) = 14.25 psf

The pressure in a city water main is 57 psi. The pressure loss through piping, fittings, and the water meter has been calculated as 23 psi, and the highest fixture requires 12 psi to operate. What is the maximum height the fixture can be above the water main? (A) 9 ft (B) 24 ft (C) 50 ft (D) 78 ft

(C) 50 ft In order to find the maximum height, first take the pressure in the water main and subtract other known pressure losses and the pressure required for the fixture to operate properly 57 psi - 23 psi - 12 psi = 22 psi Because 1 psi is required to lift water 2.3 ft, the maximum height is (22 lbf/in2) (2.3 ft/lbf/in2) = 50.6 ft

Which of the following strategies would effectively reduce the noise caused by a duct system without reducing airflow? (A) Specify duct liners for all supply and return ducts (B) Specify 90 degree bends in short duct runs (C) Provide an active noise-canceling system emitting out-of-phase noise (D) Specify fiberglass baffles

(C) Provide an active noise-canceling system emitting out-of-phase noise An active noise-canceling system would help to reduce the noise in a duct system without reducing airflow. This type of system analyzes the noise from the blowers and other system components and synthesizes a noise that is exactly out of phase with the mechanical noise. The two sounds cancel each other out and the result is perceived silence

What type of cement is used in slip form construction? (A) Type I (B) Type II (C) Type III (D) Type IV

(C) Type III Type III cement is needed for rapid slip form construction. Slip forming, continuous poured, continuously formed, or slipform construction is a construction method in which concrete is poured into a continuously moving form Type I - normal cement Type II - low heat and sulfate resistance Type III - high-early-strength Type IV - slow setting and low heat for massive structures

In a full set of construction drawings, the mechanical engineering drawings are typically placed (A) after the civil engineering drawings and before the architectural drawings (B) immediately after the architectural drawings (C) after the structural engineering drawings and before the electrical drawings (D) after the electrical drawings

(C) after the structural engineering drawings and before the electrical drawings The normal sequence of drawings in a full set of drawings is as follows: site drawings / survey civil engineering landscape architectural structural plumbing mechanical electrical

Consider the location for a mechanical system and its vertical distribution tree in a 10-story office building. Which of the following locations should be avoided? (A) within internal circulation cores (B) integrated with a building (C) at the edges of a building (D) between the internal core and the edge of a building

(C) at the edges of a building The most desirable and preferred areas for leasable office space must be considered. Exterior walls provide the best opportunity to take advantage of natural light, outside views, and natural ventilation. When mechanical systems and their distribution are placed near the center of the building, the exterior wall is made available for other uses The preferred areas are reduced when mechanical systems are placed along the outside walls at the edges of a building

In masonry walls, water is prevented from seeping back into the wall through capillary action by using (A) base flashing (B) coping (C) drips (D) weep holes

(C) drips Drips are extensions of through-wall flashing or projections below masonry units that extend beyond the primary plane of the wall. The purpose of a drip is to force water that is draining off flashing or a sill to fall down and away from the wall rather than to adhere to the wall and possibly flow back into the wall through capillary action or cracks below the flashing

A specification section written following the recommendations of the Construction Specifications Institute would include which sections? (A) general, administration, and execution (B) administration, products, and execution (C) general, products, and execution (D) general, installation, and products

(C) general, products, and execution The three-part specification format developed by the Construction Specifications Institute (CSI) includes sections entitled General, Products, Execution The General section includes administrative and procedural requirements specific to the specification The Products section includes information about materials, systems, manufactured units, shop fabrication, and factory finishing prior to installation The Execution section gives instructions for on-site incorporation of the projects into the project

Rehabilitation of a building constructed in the 1860s requires cleaning and repairing the existing brick exterior walls. The brick is in good condition but is very dirty. Portions of the mortar have fallen out, and much of what remains is so soft that it can be scraped away with a fingernail. Which restoration technique should the architect recommend? (A) pressure washing with a 10% muratic acid solution (B) pressure washing with plain water and repointing the mortar (C) hand washing the brick with water and a stiff brush and repointing the mortar (D) sandblasting

(C) hand washing the brick with water and a stiff brush and repointing the mortar The least destructive technique should always be recommended when dealing with historic buildings and fragile old building materials. Hand washing with water and a stiff brush is the gentlest cleaning method and will probably remove most of the dirt from the surface of the brick. Missing or deteriorating mortar should be removed and replaced with compatible mortar, and the mortar should be restruck to shed water from the joints. This process is called repointing or tuck pointing

Restrictions on surface finishes in all model codes are primarily based on (A) occupancy and construction type (B) occupant load and location in the building (C) location in the building and occupancy group (D) occupancy group and sprinklering

(C) location in the building and occupancy group The primary restrictions on surface finishes given in model codes (IBC) are the occupancy group and the location in the building according to exiting requirements Having a building with a sprinkler system only modifies the basic requirements and allows the required flame-spread rating to be dropped on class in some instances

A new 80,000 ft2 office building is projected to cost $10,000,000. Approximately what percentage of the construction budget should be allocated to the building's mechanical and electrical systems costs? (A) mechanical 5%, electrical 5% (B) mechanical 15%, electrical 5% (C) mechanical 15%, electrical 15% (D) mechanical 25%, electrical 15%

(C) mechanical 15%, electrical 15% For an office building of this size, the total mechanical and electrical costs are in the range of 30% of the total construction budget, about equally divided between the two For smaller office buildings, the total costs are in the range of 20%, also about equally divided

Dock levelers are used to (A) provide final adjustment for steel angle trim (B) keep boating piers stable in the water (C) provide adjustability for various sized trucks (D) accommodate tolerances in pouring concrete pits

(C) provide adjustability for various sized trucks Dock levelers are pieces of equipment used to allow trucks with various bed heights to be serviced from the same dock. The equipment is placed in a pit at the edge of the dock, and a steel panel is adjusted up or down to provide a ramp from the dock to the level of the truck

Which of the following materials would be used as a firestop? (A) wood stud (B) plywood (C) silicone foam (D) treated wood blocking

(C) silicone foam Firestops are materials or systems of materials that are used to seal penetrations through fire walls or smoke barriers. They are always noncombustible and may be factory built or constructed in the field. Depending on the wall type and application; mortar, mineral wool, or silicone foam would be acceptable for use as a firestop Draftstops also prevent the passage of fire and smoke but can be made of combustible materials such as treated wood blocking. They are placed between floors and at concealed spaces

The best measure to use when evaluating how well a window assembly prevents heat gain is the (A) daylight factor (B) shading coefficient (C) solar heat gain coefficient (D) window-to-wall ratio

(C) solar heat gain coefficient The solar heat gain coefficient is the ratio of solar heat gain through a window to the amount of solar radiation striking the window. Because it includes the frame and the glass spacer, the solar heat gain coefficient is a better indicator of heat gain than the shading coefficient, which is a similar measure but does not include the effects of the frame The daylight factor and the window-to-wall ratio are not measures of heat gain A daylight factor (DF) is the ratio of the light level inside a structure to the light level outside the structure

According to model codes regarding a staircase, which of the indicated stair dimensions is a code violation? (A) handrail height of 36" (B) headroom height of 6'-8" (C) total rise height of 14' (D) handrail extension of 12"

(C) total rise height of 14' (without a landing) The maximum allowable height between landings or floors (rise height) is 12 ft. All of the other dimensions are correct. The height from the nosing to the top of a handrail must be between 34" and 38"

Which of the following are likely to occur if the drawings and specifications are not thoroughly coordinated? (Choose the four that apply.) (A) a decrease of the actual cost from the estimated cost because the contractor bid on a less expensive material shown on the drawings, although the same material was called out as a more expensive type in the specifications (B) the immediate filing of a lawsuit (C) the need for a change order during construction to account for modifications required to correct discrepancies in the two documents (D) a delay in construction (E) an increase in cost because the contractor bid the least expensive choice between two conflicting requirements when the client wanted the more expensive option (F) the architect may be held financially responsible for the omission

(C), (D), (E), (F) It is commonly believed that the specifications take precedence over the drawings, but AIA A201, Sec. 1.2.1, states that these two types of documents are considered "complementary, and what is required by one shall be as binding as if required by all." In cases where there is a discrepancy, the conflict should be brought to the architect's attention. Any time there are conflicts in the project documents, the best that can happen is a quick resolution with no change to the cost, but usually an increase in cost results and the project may be delayed as the parties deal with administrative work. Conflicts in the documents can be corrected with an addendum prior to bidding or negotiation or by change order or construction change directive after the construction contract is signed Litigation would be a last resort and other remedies would be sought and implemented before a lawsuit occured

A retaining wall 8' high is retaining free-draining silty sand. From the table, an active pressure of 45 psf/ft may be used for the silty sand. What is the horizontal load exerted on the wall? (A) 650 plf (B) 850 plf (C) 1110 plf (D) 1440 plf

(D) 1440 plf The pressure at the bottom of the wall is (8 ft)(45 psf/ft) = 360 psf The total horizontal load is P = = Pmax (height/2) = (360 psf) (8 ft/ 2) = 1440 lbm/ft It will be shown that this total load acts at the centroid of a triangle, or one-third the distance from the base.

An architect is researching folding partitions to be specified to divide a hotel ballroom into two smaller meeting rooms. Meetings will frequently be held in each of the rooms simultaneously, and the presenters often use a lavaliere microphone with an amplification system. The hotel's facilities coordinator specifies that normal speech should not be heard from the other side of the partition, but it is acceptable for loud or amplified speech to be faintly heard. Into which range should the partition's STC rating fall? (A) 25-30 (B) 30-35 (C) 35-40 (D) 40-45

(D) 40-45 An STC rating of 40-45 would provide the desired sound level on the opposite side of the partition wall

A flat reinforced concrete slab is supported on columns only. The column grid is 24'x24'. The tributary area of a typical interior column is approximately (A) 140 ft2 (B) 290 ft2 (C) 390 ft2 (D) 580 ft2

(D) 580 ft2 The tributary area of a typical interior column in this grid is 24'x24' which is 576 ft2 290 ft2 is the approximate tributary area of a typical exterior column in this grid, this area would be 12'x24'

In the MasterFormat system, the requirements for testing a plumbing system would be located in (A) a section of Division 1 of the specifications (B) Part 1 of Section 22 40 00, Plumbing (C) Part 2 of Section 22 40 00, Plumbing (D) Part 3 of Section 22 44 00, Plumbing

(D) Part 3 of Section 22 44 00, Plumbing Requirements for testing of installed systems are in Part 3 of each technical section. Note that testing requirements for individual products and materials are in Part 2

During the final stages of contract document development, the electrical engineer discovers that mechanical ductwork is shown on the mechanical engineering drawings in a position that interferes with electrical conduit. The person responsible for resolving this conflict is the (A) electrical engineer (B) mechanical engineer (C) contractor (D) architect

(D) architect The architect is responsible for the overall coordination of all consultants' drawings and for resolving disputes and conflicts. The electrical engineer should bring the conflict to the architect's attention. The architect can then coordinate with both consultants to resolve the conflict

Which of the following types of diaphragms has the largest allowable shear value? (A) blocked plywood diaphragm with a 3/8" panel of structural I-grade (B) unblocked plywood diaphragm with a 3/8" panel of structural I-grade (C) diaphragm made of steel deck with a 2" concrete topping (D) diaphragm that consists of a 6" concrete slab

(D) diaphragm that consists of a 6" concrete slab A 6" concrete slab would have an allowable shear of about 10,000 lbg/ft A diaphragm made of steel deck has an allowable shear between 100 lbf/ft and 2,600 lbf/ft A diaphragm made of plywood generally has an allowable shear between 100lbf/ft and 800 lbf/ft

Which of the following statements is true about Division 1 specification sections? The specification sections (A) establish duties of the owner and contractor (B) define legal rights of the parties (C) establish duties of the architect (D) establish administrative procedures for the project

(D) establish administrative procedures for the project The general, supplementary, and special conditions, in addition to AIA Document A101, establish the legal relationship between the two parties and define their rights and duties. The architect's duties in administering the construction contract are also defined by these documents The administrative procedures for a project are established in Division 1 of the specifications, General Requirements. These sections explain exactly how the contractor is to perform the tasks assigned in the general, supplementary, and special conditions

Which type of lamp typically has the longest life span (A) incandescent (B) fluorescent (C) metal halide (D) high-pressure sodium

(D) high-pressure sodium A high-pressure sodium light bulb can be expected to last around 24,000 hours. A mercury vapor lamp would have a similar life expectancy Incandescent lamps have the shortest life-span of around 2,000 hourse Fluorescent and metal halide lamps vary in life-span with respect to their wattage, but can be expected to last from 10,000-20,000 hours

Typical 4"x4" ceramic tiles used as wall covering in an office building's restrooms should be specified as (A) vitreous (B) simivitreous (C) nonvitreous (D) imprervious

(D) impervious For a commercial toilet room, it is best to use impervious tile to withstand moisture and harsh chemical cleaners. Vitrification is a process of applying heat to a tile to fuse the material and make it denser. Denser tile permits less water to be absorbed. Specifying the level of vitrification is a way of classifying tile based upon its moisture absorption rate nonvitreous -- 7-15% absorption semivitreous -- 3-7% absorption vitreous -- .05-3% absorption impervious -- 0-.05% absorption

Weatherstripping affects (A) effective temperature (B) thermodynamics (C) ventilation (D) infiltration

(D) infiltration Weatherstripping helps seal joints and cracks around doors and windows, reducing air infiltration. Air infiltration is a primary factor in heat loss, so sealing these voids improves the performance of mechanical systems and conserves energy

A joint that is typically used where an addition meets an existing building to allow the two sections to move independently of one another is called a(n) (A) control joint (B) construction joint (C) expansion joint (D) isolation joint

(D) isolation joint An isolation joint allows two sections of a building to move independently, and is typically used where an addition meets and existing building or where two different materials meet. It is also called an abutment joint

Which of the following types of metal contains the most copper? (A) austenitic stainless steel (B) Monel metal (C) Muntz metal (D) nickel silver

(D) nickel silver Stainless steel is an alloy of iron, carbon, and chromium. Austenic stainless steel, the most common type, also contains some nickel and/or manganese. This kind of steel is nonmagnetic and not heat treatable Monel is a trade name for a metal alloy of copper and nickel Muntz metal is a common alloy of 60% copper and 40% tin Nickel silver is a name given to an alloy of 65% copper, 25% zinc, and 10% nickel

A designer is planning on using a hydronic distribution system to heat a building and needs to decide which type of system will be best for efficiency and comfort control. To this end, the designer plans on supplying equal flow and fluid temperatures to each radiator to yield an efficient return. Which type of piping systems would most likely achieve the best result? (A) one-pipe system (B) two-pip direct return system (C) two-pip reverse return system (D) series perimeter loop system

(D) series perimeter loop system The series perimeter loop system has a continuous loop that is pressurized, and temperature is maintained so that individual radiators can tap into it and be controlled individually. The series perimeter loop system typically does not use steam, so chilled water for cooling can be used in the warmer seasons. The loop feeds back to the heat (or cooling) source so that individual returns are not neeted In a one-pipe system (traditional steam radiator system), it is often difficult to maintain or control temperature, especially to the more remote locations. The return of the condensed water relies heavily on gravity A two-pipe direct return system does not improve on the temperature and pressure issue of the one-pipe system for the remote radiators, but the return loop still relies on gravity A two-pipe reverse return system distributes the fluid to each of the radiators through a continuous system with individual returns from each unit. This results in a longer overall system but with better control than other types of systems

According to the American Concrete Institute's Building Code Requirements for Structural Concrete (ACI 318), what is the maximum permissible spacing of lateral ties in a reinforced concrete column? (A) 16 times the diameter of the reinforcing bars (B) 48 times the diameter of the lateral ties (C) the least dimensions of the column (D) the least of 16 times the diameter of the reinforcing bars, 48 times the diameter of the lateral ties, or the least dimension of the column

(D) the least of 16 times the diameter of the reinforcing bars, 48 times the diameter of the lateral ties, or the least dimension of the column According to ACI 318, the maximum spacing between lateral ties of a reinforced concrete column should be the least of the three dimensions listed above

What type of drawings would contain finish floor elevations, information about materials and types of construction assemblies, and window head heights? (A) floor plans (B) finish schedules (C) interior elevations (D) wall sections

(D) wall sections Wall sections are vertical "slices" of the exterior wall of a building that graphically depict how the building is to be built. They show vertical dimensions, materials, and relationships and how the pieces are intended to fit together

Which of the following materials has the smallest dimensional tolerances? (A) precast concrete tees (B) 2x4 wood framing at a window opening (C) steel beams (D) wood paneling

(D) wood paneling Wood paneling has the most restrictive tolerances, as even small deviations from the prescribed dimensions are noticeable

Assuming the soil has an equivalent fluid weight of 15 psf per foot of height, what is the total earth pressure load on a retaining wall 9.5 ft high?

1354 plf P = = (15 lbf/ft3)(9.5 ft)^2 = 1354 plf

What is the maximum unsupported height for a cavity wall consisting of a 4" brick facing and a 8" concrete block wall separated by a 1" space?

216" or 18'-0" In computing the ratio for cavity walls, the thickness is the sum of the nominal thickness of the wythes. The maximum height is (4" + 8") (18) = 216"


Conjuntos de estudio relacionados

Foundations in Personal Finance Ch. 10 - T/F; Fill in blank

View Set

Organizational behavior and magnet hospitals

View Set